Breast Flashcards

1
Q

Which of the following statements about normal breast anatomy is true?

A. The breast typically contains 10 lobes.

B. Cooper ligaments are only found in the upper quadrants of the breast.

C. The upper inner quadrant of the breast contains the most breast tissue.

D. The tail of Spence extends across the anterior axillary fold.

A

D

The breast is composed of 15 to 20 lobes, which are each composed of several lobules.

Fibrous bands of connective tissue travel through the breast (Cooper suspensory ligaments), insert perpendicularly into the dermis, and provide structural support.

The mature female breast extends from the level of the second or third rib to the inframammary fold at the sixth or seventh rib. It extends transversely from the lateral border of the sternum to the anterior axillary line.

The deep or posterior surface of the breast rests on the fascia of the pectoralis major, serratus anterior, and external oblique abdominal muscles, and the upper extent of the rectus sheath.

The retromammary bursa may be identified on the posterior aspect of the breast between the investing fascia of the breast and the fascia of the pectoralis major muscles.

The axillary tail of Spence extends laterally across the anterior axillary fold.

(See Schwartz 10th ed., p. 500.)

How well did you know this?
1
Not at all
2
3
4
5
Perfectly
2
Q

Which of the following changes in the breast is NOT associated with pregnancy?

A. Accumulation of lymphocytes, plasma cells, and eosinophils within the breast.

B. Enlargement of breast alveoli.

C. Release of colostrum.

D. Accumulation of secretory products in minor duct lumina.

A

Answer: C

With pregnancy, the breast undergoes proliferative and developmental maturation.

As the breast enlarges in response to hormonal stimulation, lymphocytes, plasma cells, and eosinophils accumulate within the connective tissues. The minor ducts branch and alveoli develop.

Development of the alveoli is asymmetric, and variations in the degree of development may occur within a single lobule.

With parturition, enlargement of the breasts occurs via hypertrophy of alveolar epithelium and accumulation of secretory products in the lumina of the minor ducts. Alveolar epithelium contains abundant endoplasmic reticulum, large mitochondria, Golgi complexes, and dense lysosomes.

Two distinct substances are produced by the alveolar epithelium:
(1) the protein component of milk, which is synthesized in the endoplasmic reticulum (merocrine secretion); and

(2) the lipid component of milk (apocrine secretion),which forms as free lipid droplets in the cytoplasm.

Milk released in the first few days after parturition is called colostrum and has low lipid content but contains considerable quantities of antibodies.

(See Schwartz 10th ed.,p.501.)

How well did you know this?
1
Not at all
2
3
4
5
Perfectly
3
Q

The breast receives its blood supply from all of the following EXCEPT:

A. Branches of the internal mammary artery

B. Branches of the superior epigastric artery

C. Branches of the posterior intercostal arteries

D. Branches of thoracoacromial artery

A

Answer: B

The breast receives its principal blood supply from:

(1) perforating branches of the internal mammary artery;
(2) lateral branches of the posterior intercostal arteries; and
(3) branches from the axillary artery, including the highest thoracic, lateral thoracic, and pectoral branches of the thoracoacromial artery.

The second, third, and fourth anterior intercostal perforators and branches of the internal mammary artery arborize in the breast as the medial mammary arteries.

The lateral thoracic artery gives off branches to the serratus anterior, pectoralis major and pectoralis minor, and subscapularis muscles. It also gives rise to lateral mammary branches.

(See Schwartz 10th ed., p. 501.)

How well did you know this?
1
Not at all
2
3
4
5
Perfectly
4
Q

Which of the following statements is INCORRECT?

A. Level I lymph nodes are those that are lateral to the pectoralis minor muscle.

B. Level II lymph nodes are located deep to the pectoralis minor muscle.

C. Level III lymph nodes are located medial to the pectoralis minor muscle.

D. Level lV lymph nodes are the ipsilateral internal mammary lymph nodes.

A

Answer: D

Level I includes lymph nodes located lateral to the pectora- lis minor muscle;

Level II includes lymph nodes located deep to the pectoralis minor; and

Level III includes lymph nodes located medial to the pectoralis minor.

(See Schwartz 10th ed., p. 502.)

How well did you know this?
1
Not at all
2
3
4
5
Perfectly
5
Q

Concerning breast development before and during pregnancy, which hormonal activity pairing is INCORRECT?

A. Estrogen: Initiates ductal development

B. Progesterone: Initiates lobular development

C. Prolactin: Initiates lactogenesis

D. Follicle stimulating hormone: Cooper ligament relaxation

A

Answer: D

Estrogen initiates ductal development, whereas progesterone is responsible for differentiation of epithelium and for lobular development.

Prolactin is the primary hormonal stimulus for lactogenesis in late pregnancy and the postpartum period.

The gonadotropins luteinizing hormone (LH) and follicle- stimulating hormone (FSH) regulate the release of estrogen and progesterone from the ovaries.

In turn, the release of LH and FSH from the basophilic cells of the anterior pituitary is regulated by the secretion of gonadotropin-releasing hormone (GnRH) from the hypothalamus.

(See Schwartz 10th ed., pp. 503–504.)

How well did you know this?
1
Not at all
2
3
4
5
Perfectly
6
Q

Concerning gynecomastia, which of the following is true?

A. During senescence gynecomastia is usually unilateral.

B. During puberty gynecomastia is usually bilateral.

C. Is not associated with breast cancer except in Ehlers-Danlos patients.

D. Is classified as per a three-grade system.

A

Answer: D

In gynecomastia, the ductal structures of the male breast enlarge, elongate, and branch with a concomitant increase in epithelium.

During puberty, the condition often is unilateral and typically occurs between ages 12 and 15.

In contrast, senescent gynecomastia is usually bilateral.

Gynecomastia generally does not predispose the male breast to cancer. However, the hypoandrogenic state of Klinefelter syndrome (XXY), in which gynecomastia is usually evident, is associated with an increased risk of breast cancer.

Gynecomastia is graded based on the degree of breast enlargement, the position of the nipple with reference to the inframammary fold and the degree of breast ptosis and skin redundancy:

Grade I: mild breast enlargement without skin redundancy;

Grade IIa: moderate breast enlargement without skin redundancy;

Grade IIb: moderate breast enlargement with skin redundancy; and

Grade 3: marked breast enlargement with skin redundancy and ptosis.

(See Schwartz 10th ed., p. 505.)

How well did you know this?
1
Not at all
2
3
4
5
Perfectly
7
Q

Inflammatory conditions of the breast include all of the following EXCEPT

A. Necrotizing viral mastitis

B. Zuska disease (recurrent preductal mastitis)

C. Mondor disease (superficial breast thrombophlebitis)

D. Hidradenitis suppurativa

A

Answer: A

Zuska disease, also called recurrent periductal mastitis, is a condition of recurrent retroareolar infections and abscesses.

Hidradenitis suppurativa of the nipple-areola complex or axilla is a chronic inflammatory condition that originates within the accessory areolar glands of Montgomery or within the axillary sebaceous glands.

Mondor disease is a variant of thrombophlebitis that involves the superficial veins of the anterior chest wall and breast.

(See Schwartz 10th ed., pp. 506–507.)

How well did you know this?
1
Not at all
2
3
4
5
Perfectly
8
Q

Lesions with malignant potential include all of the following EXCEPT

A. Intraductal papilloma

B. Atypical ductal hyperplasia

C. Sclerosing adenosis

D. Atypical lobular hyperplasia

A

Answer: C

Sclerosing adenosis is prevalent during the childbearing and perimenopausal years and has no malignant potential.

Multiple intraductal papillomas, which occur in younger women and are less requently associated with nipple discharge, are susceptible to malignant transformation.

Individuals with a diagnosis of atypical ductal hyperplasia (ADH) are at increased risk or development of breast cancer and should be counseled appropriately regarding risk reduction strategies.

Atypical lobular hyperplasia (ALH) results in minimal distention of lobular units with cells that are similar to those seen in lobular carcinoma in situ (LCIS).

(See Schwartz 10th ed., pp. 509–510.)

How well did you know this?
1
Not at all
2
3
4
5
Perfectly
9
Q

Risk factors for the development of breast cancer include the following, except:

A. Early menarche

B. Nulliparity

C. Late menopause

D. Longer lactation period

A

Answer: D

Increased exposure to estrogen is associated with an increased risk for developing breast cancer, whereas reducing exposure is thought to be protective.

Correspondingly, factors that increase the number of menstrual cycles, such as early menarche, nulliparity, and late menopause, are associated with increased risk.

Moderate levels of exercise and a longer lactation period, actors that decrease the total number of menstrual cycles, are protective.

(See Schwartz 10th ed., p. 511.)

How well did you know this?
1
Not at all
2
3
4
5
Perfectly
10
Q

Drugs useful in breast cancer prevention include the following except

A. Raloxifene

B. Tamoxifen

C. Aspirin

D. Aromatase inhibitors

A

Answer: C

The P-2 trial, the Study of Tamoxifen and Raloxifene (known as the STAR trial), randomly assigned 19,747 postmenopausal women at high risk or breast cancer to receive either tamoxifen or raloxifene.

The initial report of the P-2 trial showed the two agents were nearly identical in their ability to reduce breast cancer risk, but raloxifene was associated with a more
favorable adverse event profile.

An updated analysis revealed that raloxifene maintained 76% of the efficacy of tamoxifen in prevention of invasive breast cancer with a more favorable side-effect profile.

Aromatase inhibitors (AIs) have been shown to be more effective than tamoxifen in reducing the incidence of contralateral breast cancers in postmenopausal women receiving AIs for adjuvant treatment of invasive breast cancer.

(See Schwartz 10th ed., p. 514.)

How well did you know this?
1
Not at all
2
3
4
5
Perfectly
11
Q

Which of the following is true regarding breast cancer metastasis?

A. Metastases occur after breast cancers acquire their own blood supply.

B. Batson plexus facilitates metastasis to the lung.

C. Natural killer cells have no role in breast cancer
immunosurveillance.

D. Twenty percent of women who develop breast carcinoma metastases will do so within 60 months of treatment.

A

Answer: A

At approximately the 20th cell doubling, breast cancers acquire their own blood supply (neovascularization).

Thereafter, cancer cells may be shed directly into the systemic venous blood to seed the pulmonary circulation via the axillary and intercostal veins or the vertebral column via Batson plexus of veins, which courses the length of the vertebral column.

These cells are scavenged by natural killer lymphocytes and macrophages. Sixty percent of the women who develop distant metastases will do so within 60 months of treatment.

(See Schwartz 10th ed., p. 518.)

How well did you know this?
1
Not at all
2
3
4
5
Perfectly
12
Q

All of the following are true concerning breast LCIS EXCEPT

A. Develops only in the female breast.

B. Cytoplasmic mucoid globules are a distinctive cellular feature.

C. Frequency of LCIS cannot be reliably determined.

D. The average age at diagnosis is 65 to 70 years.

A

Answer: D

LCIS originates from the terminal duct lobular units and develops only in the female breast. Cytoplasmic mucoid globules are a distinctive cellular feature.

The frequency of LCIS in the general population cannot be reliably determined because it usually presents as an incidental finding.

The average age at diagnosis is 45 years, which is approximately 15 to 25 years younger than the age at diagnosis or invasive breast cancer.

(See Schwartz 10th ed., p. 519.)

How well did you know this?
1
Not at all
2
3
4
5
Perfectly
13
Q

Which of the following concerning breast cancer staging is correct?

A. Stage I tumors have no metastases to either lymph nodes or distant sites.

B. Stage III tumors include some with distant metastases (M1 disease).

C. Inflammatory carcinoma is considered stage 4 disease.

D. N4 disease includes metastases to highest contralateral axillary nodes.

A

Answer: C

How well did you know this?
1
Not at all
2
3
4
5
Perfectly
14
Q

Factors that determine the type of therapy offered to patients after diagnosis o breast cancer include all of the following except

A. Whether or not a therapy has been proven effective in clinical trials

B. Stage of disease

C. General health of patient

D. Biologic subtype

A

Answer: A

Once a diagnosis of breast cancer is made, the type of therapy offered to a breast cancer patient is determined by the stage of the disease, the biologic subtype, and the general health status of the individual.

(See Schwartz 10th ed., p. 536.)

How well did you know this?
1
Not at all
2
3
4
5
Perfectly
15
Q

Which of the following statements about the management of ductal carcinoma in situ (DCIS) is true?

A. DCIS treated by mastectomy has a local recurrence rate of 2%.

B. Extensive DCIS should be treated with tamoxien followed by lumpectomy.

C. Specimen mammography is only useful for patients with small amounts of DCIS.

D. Postoperative tamoxifen is useful in DCIS patients whose tumors are estrogen-receptor (ER) negative.

A

Answer: A

Women with DCIS and evidence of extensive disease (>4 cm of disease or disease in more than one quadrant) usually require mastectomy.

For women with limited disease, lumpectomy and radiation therapy are generally recommended.

For nonpalpable DCIS, needle localization or other image-guided techniques are used to guide the surgical resection.

Specimen mammography is performed to ensure that all visible evidence of cancer is excised.

Adjuvant tamoxifen therapy is considered for DCIS patients with estrogen-receptor (ER)-positive disease.

The gold standard against which breast conservation therapy or DCIS is evaluated is mastectomy.

Women treated with mastectomy have local recurrence and mortality rates of <2%.

(See Schwartz 10thed.,p.537.)

How well did you know this?
1
Not at all
2
3
4
5
Perfectly
16
Q

All of the following are true about accelerated partial breast irradiation (APBI) EXCEPT

A. APBI is delivered in an abbreviated fashion and a
lower total dose than standard course of whole breast radiation.

B. Suitable patients for APBI include women older than
or equal to 60 years.

C. Suitable patients for APBI include patients whose
tumor margins are greater than or equal to 2 mm.

D. Suitable patients for APBI include those with multifocal disease.

A

Answer: D

Accelerated partial breast irradiation (APBI) is delivered in an abbreviated fashion (twice daily or 5 days) and at a lower total dose compared with the standard course of 5 to 6 weeks of radiation (50 Gray with or without a boost) in the case of whole breast irradiation.

The ASTRO guidelines describe patients “suitable” for APBI to include women older than 60 years with a unifocal, T1, ER-positive tumor with no lymphovascular invasion, and margins of at least 2 mm.

Finally, a group felt to be “unsuitable” for APBI includes those with
3 or 4 disease, ER-negative disease, multifocality, multicentricity, extensive lymphovascular invasion (LVI), or positive margins.

(See Schwartz 10th ed., p. 539.)

How well did you know this?
1
Not at all
2
3
4
5
Perfectly
17
Q

Patients not suitable for sentinel lymph node (SLN) biopsy include all of the following EXCEPT

A. Inflammatory carcinoma of the breast.

B. Prior axillary surgery.

C. Biopsy-proven distant metastases.

D. Lower inner quadrant of breast primary carcinoma.

A

Answer: D

Clinical situations where sentinel lymph node (SLN) dissection is not recommended include patients with inflammatory breast cancers, those with palpable axillary lymphadenopathy and biopsy-proven metastasis, DCIS without mastectomy, or prior axillary surgery.

Although limited data are available, SLN dissection appears to be safe in pregnancy when performed with radioisotope alone.

(See Schwartz 10th ed., p. 545.)

How well did you know this?
1
Not at all
2
3
4
5
Perfectly
18
Q

Which of the following is true concerning breast cancer during pregnancy?

A. Metastases to lymph nodes occur in approximately
75%o patients.

B. Approximately 50% of breast nodules developing
during pregnancy are malignant.

C. Mammography is especially useful in localizing small lesions.

D. There is risk of chemotherapy teratogenicity if used during the second, but not the third, trimester of pregnancy.

A

Answer: A

Breast cancer occurs in 1 of every 3000 pregnant women, and axillary lymph node metastases are present in up to 75% of these women.

Fewer than 25% of the breast nodules developing during pregnancy and lactation will be cancerous.

Mammography is rarely indicated because of its decreased sensitivity during pregnancy and lactation; however, the fetus can be shielded if mammography is needed.

Chemotherapy administered during the first trimester carries a risk of spontaneous abortion and a 12% risk of birth defects.

There is no evidence of teratogenicity resulting from administration of chemotherapeutic agents in the second and third trimesters.

(See Schwartz 10th ed., p. 554.)

How well did you know this?
1
Not at all
2
3
4
5
Perfectly
19
Q

Embryology of the breast?

A

At 5th-6th week AOG, 2 ventral bands of thickened ectoderm (mammary ridges and milk lines) are evident in the embryo. In most mammals, paired breasts develop along these ridges, which extend from the base of the forelimb (future axilla) to the region of the hind limb (inguinal area).

Accessory breasts (polymastia) or accessory nipples (polythelia) may occur along the milk line when normal regression fails. Each breast develops when an ingrowth of ectoderm forms a primary tissue bud in the mesenchyme. The primary bud, in turn, initiates the development of 15-20 secondary buds. Epithelial cords develop from the secondary buds and extend into the surrounding mesenchyme. Major (lactiferous) ducts develop, which open into a shallow mammary pit. During infancy, a proliferation of mesenchyme transforms the mammary pit into a nipple.

If there is failure of a pit to elevate above skin level, an inverted nipple results. This congenital malformation occurs in 4% of infants.

At birth, the breasts are identical in males and females, demonstrating only the presence of major ducts. Enlargement of the breast may be evident, and a secretion (“witch’s milk”) may be produced. These transitory events occur in response to maternal hormones that cross the placenta.

How well did you know this?
1
Not at all
2
3
4
5
Perfectly
20
Q

Discuss amastia.

A

Amastia is absence of the breast, which is rare and results from an arrest in mammary ridge development that occurs during the sixth fetal week.

How well did you know this?
1
Not at all
2
3
4
5
Perfectly
21
Q

Discuss Poland’s syndrome.

A

Poland’s syndrome consists of hypoplasia or complete absence of the breast, costal cartilage and rib defects, hypoplasia of the subcutaneous tissues of the chest wall, and brachysyndactyly.

Breast hypoplasia also may be iatrogenically induced before puberty by trauma, infection, or radiation therapy.

How well did you know this?
1
Not at all
2
3
4
5
Perfectly
22
Q

Discuss symmastia.

A

Symmastia is a rare anomaly recognized as webbing between the breasts across the midline.

How well did you know this?
1
Not at all
2
3
4
5
Perfectly
23
Q

How are staphylococcal breast abscesses managed versus streptoccocal breast abscesses?

A

Staphyloccocal breast abscesses are more often deep and localized. Initial management is antibiotics and repeated aspiration. Operative drainage is reserved for:

1) cases that do NOT resolve with repeated aspiration and antibiotics
2) other indications for I&D (thinning or necrosis of overlying skin)
3) biopsy of abscess cavity wall should be considered during I&D to rule breast CA in patients where antibiotics and drainage have been ineffective.

Streptococcal breast abscesses are more often diffuse. Initial management is IV antibiotics (penicillins or cephalosporins) and warm compress.

How well did you know this?
1
Not at all
2
3
4
5
Perfectly
24
Q

How do you manage noninfectious inflammation or milk stasis?

A

Epidemic puerperal mastitis is initiated by highly virulent strains of MRSA, transmitted via the suckling neonate. Purulent fluid may be expressed from the nipple.
Tx: Breastfeeding is stopped, antibiotics are started, and surgical therapy is initiated.

Nonepidemic (sporadic) puerperal mastitis refers to involvement of the interlobular connective tissue of the breast by an infectious process. The patient develops milk stasis and nipple fissuring, which initiates a retrograde bacterial infection.
Tx: Emptying of the breast using suction pumps shortens the duration of symptoms and reduces the incidence of recurrences. The addition of antibiotic therapy results in a satisfactory outcome in >95% of cases.

How well did you know this?
1
Not at all
2
3
4
5
Perfectly
25
Q

How do you manage Zuska’s disease?

A

Zuska’s disease, AKA recurrent periductal mastitis, is a condition of recurrent retroareolar infections and abscesses. Smoking has been implicated as a risk factor for this condition.

Tx: This is managed symptomatically with antibiotics coupled with incision and drainage as needed.

How well did you know this?
1
Not at all
2
3
4
5
Perfectly
26
Q

How do you manage mycotic infections of the breast?

A

Fungal infections of the breast are rare, and involve blastomycosis or sporotrichosis.

Intraoral fungi that are inoculated into the breast tissue by the suckling infant initiate these infections, which present as mammary abscesses in close proximity to the nipple-areola complex. pus mixed with blood may be expressed from sinus tracts.

Tx: Antifungal agents can be given for systemic (noncutaneous) infections.

Candida albicans affecting the skin of the breast presents as erythematous, scaly lesions of the inframammary or axillary folds. Scrapings show fungal elements (filaments and binding cells). Therapy involves removal of predispoing factors (maceration) and applying topical nystatin.

How well did you know this?
1
Not at all
2
3
4
5
Perfectly
27
Q

How do you manage Hidradenitis suppurativa?

A

Hidradenitis suppurativa is a chronic inflammatory condition that originates within the accessory areolar glands of Montgomery (NAC) or within sebaceous glands (axilla).

Women with chronic acne are predisposed.

May mimic Paget’s disease or invasive breast cancer. Often multifocal and contiguous in the axilla.

Tx: Antibiotics therapy with incision and drainage of fluctuant areas.
Excision of involved areas may be required– consider flaps and STSG for large areas of skin loss.

How well did you know this?
1
Not at all
2
3
4
5
Perfectly
28
Q

How do you manage Mondor’s disease?

A

Mondor’s disease AKA string phlebitis is a variant of thrombophlebitis that involves the superficial veins of the anterior chest wall and abreast. It involves a thrombosed vein presenting as a tender, cord-like structure.

Common veins include:
Lateral thoracic vein
Thoracoepigastric vein
Superficial epigastric vein

This is a benign and self-limiting disorder (4-6 weeks). But for an uncertain diagnosis or the presence of a mass adjacent to the tender cord, biopsy is indicated.

Tx: Anti-inflammatory meds
Warm compresses
When sx persist/refractory to tx, consider excision of involved vein segment.

How well did you know this?
1
Not at all
2
3
4
5
Perfectly
29
Q

ANDI Classification of Benign Breast Disorders for Early Reproductive Years?

A

Early Reproductive Years
15-25 years old

NORMAL
Lobular development
Stromal development
Nipple eversion
*Small fibroadenoma ≤1cm

DISORDER
Fibroadenoma ≤3cm
Adolescent hypertrophy
Nipple inversion

DISEASE
Giant fibroadenoma >3cm
Multiple fibroadenomas >5 in  one breast
Gigantomastia
Subareolar abscess
Mammary duct fistula
How well did you know this?
1
Not at all
2
3
4
5
Perfectly
30
Q

ANDI Classification of Benign Breast Disorders for Later Reproductive Years?

A

Later reproductive years
25-40 years old

NORMAL
Cyclical changes of menstruation
Epithelial hyperplasia of pregnancy

DISORDER
Cyclical mastalgia
Painful nodularity >1 week of the menstrual cycle
Bloody nipple discharge

DISEASE
Incapacitating mastalgia

How well did you know this?
1
Not at all
2
3
4
5
Perfectly
31
Q

ANDI Classification of Benign Breast Disorders for Involution?

A

Involution
35-55 years old

NORMAL
Lobular involution
Duct involution (Dilatation, Sclerosis)
Epithelial turnover

DISORDER
Macrocysts
Sclerosing lesions
Duct ectasia
Nipple retraction
Epithelial hyperplasia

DISEASE
Periductal mastitis
Epithelial hyperplasia with atypia

How well did you know this?
1
Not at all
2
3
4
5
Perfectly
32
Q

Cancer risk associated with benign breast disorders and in situ carcinoma of the breast?

A

NO RISK
Nonproliferative lesions
Sclerosing adenosis
Intraductal papilloma

1.5-2-FOLD RISK
Florid hyperplasia

4-FOLD RISK
Atypical lobular hyperplasia
Atypical ductal hyperplasia

7-FOLD RISK
Ductal involvement by cells of atypical ductal hyperplasia

10-FOLD RISK
Lobular carcinoma in-situ
Ductal carcinoma in-situ

How well did you know this?
1
Not at all
2
3
4
5
Perfectly
33
Q

Nonproliferative breast disorders?

A
NONPROLIFERATIVE
70% of Benign Breast Conditions
(No increased risk for cancer)
1) Cysts and apocrine metaplasia
2) Duct ectasia
3) Mild ductal epithelial hyperplasia
4) Calcifications
5) Fibroadenoma and related lesions
How well did you know this?
1
Not at all
2
3
4
5
Perfectly
34
Q

Calcifications associated with cancer?

A

1) Microcalcifications <0.5mm

2) Fine, linear calcifications, which may show branching

How well did you know this?
1
Not at all
2
3
4
5
Perfectly
35
Q

Proliferative breast disorders without atypia?

A

PROLIFERATIVE WITHOUT ATYPIA

1) Sclerosing adenosis
- Childbearing and perimenopausal years
- Distorted breast lobules
- In the context of multiple microcysts or a palpable mass.

2) Radial scars (≤1cm) and complex sclerosing lesions (>1cm)
- Imaging features of a radial scar will dictate the need for either a vacuum-assisted biopsy or surgical excision to exclude the possibility of a carcinoma.

3) Ductal epithelial hyperplasia
- Mild: 3 to 4 layers above the basement membrane
- Moderate: ≥5 layers
- Florid: ≥70% of a minor duct lumen (solid or papillary, 1.5-2 fold risk of CA)

4) Intraductal papillomas
- Arise in major ducts, in premenopausal women
- <0.5cm but may be as large as 5cm
- (+) nipple discharge
- attached to wall of involved duct by a stalk
- NO increased risk of CA, unless multiple

How well did you know this?
1
Not at all
2
3
4
5
Perfectly
36
Q

Differentiate adenomas/other lesions of the breast from fibroadenomas.

A

ADENOMAS
Adenomas of the breast are well-circumscribed and composed of benign epithelium with sparse stroma, which is the histologic feature that differentiates them from fibroadenomas.

They may be divided into:

1) tubular adenomas (young, nonpregnant women)
2) lactating adenomas (during pregnancy or postpartum)

HAMARTOMAS
Discrete breast tumors 2-4cm, firm, sharply circumscribed.

ADENOLIPOMAS
Sharply circumscribed nodules of fatty tissue that contain normal breast lobules and ducts.

How well did you know this?
1
Not at all
2
3
4
5
Perfectly
37
Q

Proliferative breast disorders with atypia?

A

ATYPICAL PROLIFERATIVE BREAST LESIONS
Have some of the features of carcinoma in situ, but either lack a major defining feature of carcinoma in situ or have the features in less than fully developed form.

1) Atypical ductal hyperplasia (ADH)
- appears similar to low-grade DCIS histologically
- monotonous round, cuboidal, or polygonal cells enclosed by basement membrane with rare mitoses
- ADH: ≤2-3mm
- DCIS: >3mm
- usually needs excision biopsy for diagnosis
- INCREASED RISK for development of breast CA

2) Atypical lobular hyperplasia (ALH)
- minimal distension of lobular units with cells similar to those seen in LCIS
- small monomorphic cells that distend the terminal ductal lobular unit

  • LCIS
  • in LCIS, the acini are full and distended while the overall lobular architecture is maintained).
  • classic LCIS is not associated with a specific mammographic or palpable abnormality but is an incidental finding noted on breast biopsy.
  • Pleomorphic LCIS is a variant– there can be calcifications or other mammographic changes that dictate the need for biopsy.
  • classic LCIS is not treated with excision as the patient is at risk for developing invasive breast cancer in either breast, and therefore the patient is counseled regarding appropriate risk reduction strategies.
  • Pleomorphic LCIS can be difficult to distinguish from high-grade DCIS, and there are some proponents who have suggested that patients with pleomorphic LCIS be managed similar to those with DCIS with attention to margins and consideration for RT in the setting of breast conserving treatment.
  • the use of IHC for e-cadherin can help discriminate between LCIS and DCIS.
  • -> In lobular neoplasias, such as ALH and LCIS, there is a lack of E-cadherin expression, whereas the majority of ductal lesions will demonstrate E-cadherin activity.
How well did you know this?
1
Not at all
2
3
4
5
Perfectly
38
Q

What is the clinical significance of sclerosing adenosis? How is this managed?

A

The clinical significance of sclerosing adenosis lies in its imitation of cancer. Excisional biopsy and histologic exam are frequently necessary to exclude the diagnosis of cancer.

The diagnostic workup for radial scars and complex sclerosing lesions frequently involves stereotactic biopsy.

Usually it is not possible to differentiate these lesions with certainty from cancer by mammographic features, so a larger tissue biopsy is recommended either by way of vacuum-assisted biopsy or an open surgical excisional biopsy.

The mammographic appearance of a radial scar or sclerosing adenosis (mass density with spiculated margins) will usually lead to an assessment that the results of a CNB specimen showing benign disease are discordant with radiography findings.

How well did you know this?
1
Not at all
2
3
4
5
Perfectly
39
Q

What is the difference between puerperal abscesses and subareolar abscesses?

A

Unlike puerperal abscesses, a subareolar abscess is usually unilocular and often is associated with a single duct system. Ultrasound will accurately delineate its extent.

Recurrent abscess with fistula is treated by fistulectomy and primary closure with antibiotic coverage (ie, young women with squamous metaplasia of a single duct).

When subareolar sepsis is diffuse rather than localized, or more than one fistula is present, total duct excision is the most expeditious approach (ie, older women with multiple ectatic ducts).

Antibiotic therapy is useful for recurrent infection after fistula excision, and a 2-4 weeks course is recommended before total duct excision.

How well did you know this?
1
Not at all
2
3
4
5
Perfectly
40
Q

Discuss nipple inversion.

A

More women request correction of congenital nipple inversion than for nipple inversion secondary to duct ectasia.

Although the results are usually satisfactory, women seeking correction for cosmetic reasons should always be made aware of the surgical complications of altered nipple sensation, necrosis, and fibrosis with nipple retraction.

Nipple inversion is a result of shortening of the subareolar ducts, hence a complete division of these ducts is necessary for permanent correction of the disorder.

How well did you know this?
1
Not at all
2
3
4
5
Perfectly
41
Q

Indications for FISTULECTOMY in recurrent subareolar sepsis?

A

1) Small abscess localized to one segment
2) Recurrence involving the same segment
3) Mild or no nipple inversion
4) Patient unconcerned about nipple inversion
5) Younger patient
6) No discharge from other ducts
7) No prior fistulectomy

How well did you know this?
1
Not at all
2
3
4
5
Perfectly
42
Q

Indications for TOTAL DUCT EXCISION in recurrent subareolar sepsis?

A

1) Large abscess affecting >50% of the areolar circumference
2) Recurrence involving a different segment
3) Marked nipple inversion
4) Patient requests correction of nipple inversion
5) Older patient
6) Purulent discharge from other ducts
7) Recurrence after fistulectomy

How well did you know this?
1
Not at all
2
3
4
5
Perfectly
43
Q

Hormonal and nonhormonal risk factors for breast cancer?

A

HORMONAL

1) Increased exposure to estrogen:
- Early menarche
- Nulliparity
- Late menopause

2) Older age at first live birth

3) Obesity
(Major source of estrogen in postmenopausal women is conversion of androstenedione to estrone by adipose tissue; hence obesity is associated with a long term increase in estrogen exposure.

NON-HORMONAL
1) Radiation exposure (esp during adolescence, a period of active breast development)

2) Alcohol intake (increases levels of estradiol)
3) Fatty food intake (increases serum estrogen levels)

How well did you know this?
1
Not at all
2
3
4
5
Perfectly
44
Q

Protective factors against breast cancer?

A

Factors that decrease the total number of menstrual cycles:

1) Moderate levels of exercise
2) Longer lactation period
3) Terminal differentiation of breast epithelium associated with a full-term pregnancy

How well did you know this?
1
Not at all
2
3
4
5
Perfectly
45
Q

Factors in the Gail Model for Risk Assessment in Breast Cancer?

A

1) Age
2) Age at menarche
3) Age at first live birth
4) Number of breast biopsy specimens
5) Any history of atypical hyperplasia
6) Number of first-degree relatives with breast cancer

A woman’s risk factors are translated into an overall risk score by multiplying her relative risks from several categories. This risk score is then compared to an adjusted population risk of breast cancer to determine a woman’s individual or absolute risk.

The output is a 5-year risk and lifetime risk of developing breast cancer.

How well did you know this?
1
Not at all
2
3
4
5
Perfectly
46
Q

How does the Claus model of breast cancer risk work?

A

Using data from the Cancer and Steroid Hormone Study (case-control), the Claus risk assessment model is based on assumptions about the prevalence of high-penetrance breast cancer susceptibility genes.

This incorporates more information about family history but excludes other risk factors. It provides individual estimates of breast cancer risk according to decade of life based on presence of first and second degree relatives with breast cancer and their age at diagnosis.

How well did you know this?
1
Not at all
2
3
4
5
Perfectly
47
Q

How does the BRCAPRO model of breast cancer risk work?

A

The BRCAPRO model is a Mendelian model that calculates the probability that an individual is a carrier of a mutation in one of the breast cancer susceptibility genes based on their family history of breast and ovarian cancer.

This is based on age-specific incidence curves for both mutation carriers and noncarriers.

How well did you know this?
1
Not at all
2
3
4
5
Perfectly
48
Q

How does the Tyrer-Cuzick model for breast CA risk work?

A

It attempts to utilize both family history information and individual risk information. It uses family history to calculate the probability that an individual carries a mutation in one of the breast cancer susceptibility genes, and then the risk is adjusted based on personal risk factors, including:

age at menarche
parity
age at first live birth
age at menopause
history of atypical hyperplasia or LCIS
height
BMI
How well did you know this?
1
Not at all
2
3
4
5
Perfectly
49
Q

Indications for the use of MRI in breast cancer screening?

A

At least 20-25% lifetime risk based on risk assessment tools, considering:

1) Family history of breast CA
2) BRCA mutation carriers
3) Family member with a BRCA mutation who has not been tested
4) Individuals receiving radiation to the chest between 10-30 years old
5) Individuals with a history of Li-Fraumeni syndrome, Cowden syndrome, Bannayan-Riley-Ruvacalba syndrome
6) With a first degree relative with one of these syndromes

MRI is an extremely sensitive screening tool that is not limited by the density of the breast tissue as mammography is; however, its specificity is moderate, leading to more false-positive events and the increased need for biopsy.

How well did you know this?
1
Not at all
2
3
4
5
Perfectly
50
Q

Side effects of tamoxifen?

A
DVT (1.6x more often)
Pulmonary emboli (3x more often)
Endometrial cancer (2.5x more often)
How well did you know this?
1
Not at all
2
3
4
5
Perfectly
51
Q

Are aromatase inhibitors more effective than tamoxifen?

A

Aromatase inhibitors (AIs) have been shown to be more effective than tamoxifen in reducing the incidence of contralateral breast cancers in postmenopausal women receiving AIs for adjuvant treatment of invasive breast cancer.

The American Society of Clinical Oncology recommends tamoxifen for chemoprevention in premenopausal or postmenopausal women and consideration for raloxifene or exemestane in postmenopausal women who are noted to be at increased risk of breast cancer.

How well did you know this?
1
Not at all
2
3
4
5
Perfectly
52
Q

What percentage of breast cancers are caused by germ line mutation inheritance?

A

Up to 5% of breast cancers are caused by inheritance of germ line mutations such as BRCA1 and BRCA2.

These are inherited in an autosomal dominant fashion with varying degrees of penetrance. They both function as tumor suppressor genes, and for each gene, the loss of both alleles is required for the initiation of cancer.

How well did you know this?
1
Not at all
2
3
4
5
Perfectly
53
Q

Discuss BRCA1.

A

BRCA1 is located on chromosome arm 17q, spans a genomic region of approximately 100 kilobases (kb) of DNA, and contains 22 coding exons for 1863 amino acids.

Germline mutations in BRCA1 represent a predisposing genetic factor in as many as:

  • 45% of hereditary breast cancers
  • at least 80% of hereditary ovarian cancers.

Female mutation carriers have been reported to have up to an 85% lifetime risk (for some families) for developing breast cancer and up to a 40% lifetime risk for developing ovarian cancer. The average lifetime risk is between 60-70%. Approximately 50% of children of carriers inherit the trait.

In general, BRCA1-associated breast cancers are invasive ductal carcinomas, are poorly differentiated, are in the majority hormone receptor negative, and have a triple receptor negative (immunohistochemical profile: ER-negative, PR-negative, and HER2-negative) or basal phenotype (based on gene expression profiling). BRCA1-associated breast cancers have a number of distinguishing clinical features, such as an early age of onset compared with sporadic cases; a higher prevalence of bilateral breast cancer; and the presence of associated cancers in some affected individuals, specifically ovarian cancer and possibly colon and prostate cancers.

The two most common mutations are 185delAG and 5382insC, which account for 10% of all the mutations seen in BRCA1. These two mutations occur at a 10-fold higher frequency in the Ashkenazi Jewish population than in non-Jewish Caucasians. The carrier frequency of the 185delAG mutation in the Ashkenazi Jewish population is 1% and, along with the 5382insC mutation, accounts for almost all BRCA1 mutations in this population. Analysis of germline mutations in Jewish and non-Jewish women with early-onset breast cancer indicates that 20% of Jewish women who develop breast cancer before age 40 years carry the 185delAG mutation. There are founder BRCA1 mutations in other populations including, among others, Dutch, Polish, Finnish, and Russian populations.

How well did you know this?
1
Not at all
2
3
4
5
Perfectly
54
Q

Discuss BRCA2.

A

BRCA2 is located on chromosome arm 13q, and spans a genomic region of approx. 70kb of DNA. The 11.2-kb coding region gontains 26 coding exons, and encodes a protein of 3418 amino acids.

It is postulated to play a role in DNA damage response pathways. BRCA2 messenger RNA also is expressed at high levels in the late G1 and S phases of the cell cycle.

The breast cancer risk for BRCA2 mutation carriers is close to 85%.

The lifetime ovarian cancer risk is lower than for BRCA1, but is still estimated to be close to 20%.

50% of children of carriers inherit the trait.

Unlike male carriers of BRCA, men with germline mutations in BRCA2 have an estimated breast cancer risk of 6% (100-fold increase over the risk in the general male population).

BRCA2 associated breast cancers are invasive ductal carcinomas, which are more likely to be well-differentiated and to express hormone receptors that are BRCA-1 associated.

BRCA2 associtated breast cancer has a number of distinguishing features:

  • Early age of onset
  • Higher prevalence of bilateral breast CA
  • Presence of associated cancers in some affected individuals (ovarian, colon, prostate, pancreatic, GB, bile duct, stomach, melanoma)

Founder mutations identified in BRCA2 include:

  • 6174delT mutation (Ashkenazi Jews) - 1.2% (60% of ovarian CA and 30% of early onset breast CA)
  • 999del5 - Icelandic and Finnish populations
How well did you know this?
1
Not at all
2
3
4
5
Perfectly
55
Q

What is the four-step process to identifying BRCA Mutation Carriers?

A
  1. Obtain a complete multigenerational family history
  2. Assess appropriateness of genetic testing for a particular patients
  3. Counsel the patient
  4. Interpret testing results

Genetic testing should be offered only in conjunction with patient education and counseling, including referral to a genetic counselor.

A thorough and accurate family history is crucial, because 50% of women with a BRCA mutation have inherited the mutation from their fathers.

How well did you know this?
1
Not at all
2
3
4
5
Perfectly
56
Q

When is a hereditary risk of breast cancer considered?

A

If a family includes:

1) Ashkenazi Jewish heritage
2) A first degree relative with breast cancer before age 50
3) A history of ovarian cancer at any age in the patient or first or second degree relative with ovarian cancer
4) Breast and ovarian cancer in the same individual
5) 2 or more first or second degree relatives with breast cancer at any age
6) Patient or relative with bilateral breast cancer
7) Male breast cancer in a relative at any age

The threshold for genetic testing is lower in individuals who are members of ethnic groups in whom the mutation prevalence is increased.

How well did you know this?
1
Not at all
2
3
4
5
Perfectly
57
Q

What test is clinically available for analyzing BRCA mutations?

A

Gene sequence analysis

How well did you know this?
1
Not at all
2
3
4
5
Perfectly
58
Q

What is phenocopy?

A

A familial cancer is caused by an identifiable BRCA mutation, but an individual tested had sporadic cancer.

This is possible if the individual tested developed breast cancer close to the age of onset of the general population (age 60 years or older) rather than before age 50 years, as is characteristic of BRCA mutation carriers.

How well did you know this?
1
Not at all
2
3
4
5
Perfectly
59
Q

What is the false-negative rate for BRCA mutation testing?

A

<5%

How well did you know this?
1
Not at all
2
3
4
5
Perfectly
60
Q

What are the risk management strategies for BRCA1 and BRCA2 mutation carriers?

A

1) Risk-reducing mastectomy and reconstruction
2) Risk-reducing salpingo-oophorectomy
3) Intensive surveillance for breast and ovarian cancer
4) Chemoprevention

How well did you know this?
1
Not at all
2
3
4
5
Perfectly
61
Q

What are the screening recommendations for BRCA mutation carriers who do not undergo risk-reducing mastectomy?

A

1) Clinical breast examination every 6 months

2) Mammography every 12 months beginning at age 25
(The risk of breast CA in carriers increases after age 30)

3) Annual MRI in women with a 20-25% or greater lifetime risk of developing breast cancer (mainly based on family history), women with a known BRCA1 or BRCA2 mutation, those with a first degree relative with a BRCA1 or BRCA2 mutation and have not had genetic testing themselves, those treated with RT to the chest between 10-30 years old, and those with Li-Fraumeni/Cowden/Bannayan-Riley-Ruvacalba syndrome, or a first degree relative with one of these syndromes.

How well did you know this?
1
Not at all
2
3
4
5
Perfectly
62
Q

What is the risk of ovarian cancer in BRCA1 and BRCA2 mutation carriers?

A

20-40% (10x higher than in the general population)

Risk-reducing salpingo-oophorectomy is a reasonable prevention option in mutation carriers. In women with a documented BRCA1 or BRCA2 mutation, consideration for bilateral risk-reducing salpingo-oophorectomy should be between 35-40 years old, and at the completion of childbearing.

Hormone replacement therapy is discussed with the patient at the time of oophorectomy.

How well did you know this?
1
Not at all
2
3
4
5
Perfectly
63
Q

What are recommendations for screening for mutation carriers who opt to defer risk-reducing surgery?

A

Yearly transvaginal ultrasound timed to avoid ovulation and annual measurement of serum cancer antigen 125 levels, beginning at age 25 years.

How well did you know this?
1
Not at all
2
3
4
5
Perfectly
64
Q

When do breast cancers acquire their own blood supply?

A

At approximately the 20th cell doubling, breast cancers acquire their own blood supply. Thereafter, cancer cells may be shed directly into the systemic venous blood to seed the pulmonary circulation via the axillary and intercostal veins or the vertebral column via Batson’s plexus of veins, which courses the length of the vertebral column.

How well did you know this?
1
Not at all
2
3
4
5
Perfectly
65
Q

When does implantation of metastatic foci occur in breast cancer?

A

After the primary cancer exceeds 0.5cm, which corresponds to the 27th cell doubling.

How well did you know this?
1
Not at all
2
3
4
5
Perfectly
66
Q

What is multicentricity?

A

The occurence of a second breast cancer outside the breast quadrant of the primary cancer (or at least 4cm away).

This occurs in 60-90% of women with LCIS, and 40-80% of those with DCIS.

LCIS occurs bilaterally in 50-70% of cases, while DCIS occurs bilaterally in 10-20% of cases.

How well did you know this?
1
Not at all
2
3
4
5
Perfectly
67
Q

What is multifocality?

A

The occurence of a second cancer within the same breast quadrant as the primary cancer (or within 4cm of it).

How well did you know this?
1
Not at all
2
3
4
5
Perfectly
68
Q

Discuss LCIS.

A

LCIS originates from terminal duct lobule units, and develops only in the female breast.

It is characterized by distention and distortion of the terminal duct lobular units by cells that are large, but maintain a normal nuclear to cytoplasmic ratio.

Cytoplasmic mucoid globules are a distinctive cellular feature.

Neighborhood calcification (LCIS may be observed in tissues with microcalcifications, but the calcifications associated with LCIS typically occur in adjacent tissues) is a feature unique to LCIS.

LCIS usually presents as an incidental finding, diagnosed on average at 45 years, 12x more frequently in white women.

Invasive breast cancer develops in 25-35% of women with LCIS, with 65% of subsequent cancers becoming ductal in origin. Hence LCIS is regarded as a marker of increased risk rather than as an anatomic precursor.

Individuals should be counseled regarding their risk of developing breast cancer and appropriate risk reduction strategies.

How well did you know this?
1
Not at all
2
3
4
5
Perfectly
69
Q

Discuss DCIS.

A

Predominantly seen in female breast CA, DCIS accounts for 5% of male breast cancers.

It is characterized by a proliferation of minor ductal epithelium, resulting in papillary growths within the duct lumina. These papillary growths eventually coalesce and fill the duct lumina so only scattered rounded spaces remain between the clumps of atypical cancer cells, which show hyperchromasia and loss of polarity (cribriform growth pattern).

Eventually pleomorphic cancer cells obliterate the lumina and distend the ducts (solid growth pattern).

With continued growth, these cells outstrip the blood supply and become necrotic (comedo growth pattern). Calcium deposition occurs in necrotic areas and is a common feature on mammography.

The risk for invasive breast cancer is increased nearly fivefold in women with DCIS. The invasive CA are observed in the ipsilateral breast, usually the same quadrant as the DCIS (suggesting DCIS as an anatomic precursor).

How well did you know this?
1
Not at all
2
3
4
5
Perfectly
70
Q

DCIS Classification?

A

1) COMEDO
- High nuclear grade
- Extensive necrosis
- High DCIS grade

2) INTERMEDIATE
- Intermediate nuclear grade
- Focal or absent necrosis
- Intermediate DCIS grade

3) NONCOMEDO (solid, cribriform, papillary, or focal micropapillary)
- Low nuclear grade
- Absent necrosis
- Low DCIS grade

How well did you know this?
1
Not at all
2
3
4
5
Perfectly
71
Q

Discuss Paget’s disease.

A

Paget’s disease of the nipple frequently presents as a chronic, eczematous eruption of the nipple, which may be subtle but may progress to an ulcerated, weeping lesion.

It is usually associated with extensive DCIS and may be associated with an invasive cancer. A palpable mass may or may not be present. A nipple biopsy specimen will show a population of cells that are identical to the underlying DCIS cells (pagetoid features or pagetoid change).

Pathognomonic of this cancer is the presence of large, pale, vacuolated cells (Paget cells) in the rete pegs of the epithelium.

Paget’s disease may be confused with superficial spreading melanoma. Differentiation from pagetoid intraepithelial melanoma is based on the presence of S-100 antigen immunostaining in melanoma and carcinoembryonic antigen immunostaining in Paget’s disease.

Surgical therapy for Paget’s may involve lumpectomy or mastectomy, depending on the extent of involvement of the NAC and the presence of DCIS or invasive cancer in the underlying breast parenchyma.

How well did you know this?
1
Not at all
2
3
4
5
Perfectly
72
Q

Discuss invasive ductal carcinoma of the breast.

A

Invasive ductal carcinoma of the breast with productive fibrosis (scirrhous, simplex, NST), accounts for 80% of breast cancers, and presents with macroscopic or microscopic axillary lymph node mets in up to 25% of screen-detected cases, and up to 60% of symptomatic cases.

It occurs most frequently in perimenopausal or postmenopausal women in the 5th-6th decades of life as a solitary, firm mass. It has poorly defined margins, and its cut surfaces show a central stellate configuration with chalky white or yellow streaks extending into the surrounding breast tissues. The cancer cells often are arranged in small clusters and there is a broad spectrum of histologic types with variable cellular and nuclear grades.

75% of ductal cancers showed estrogen receptor expression.

How well did you know this?
1
Not at all
2
3
4
5
Perfectly
73
Q

Discuss medullary carcinoma of the breast.

A

Medullary cancer is a special type breast cancer, which accounts for 4% of all invasive breast cancers and is a frequent phenotype of BRCA1 hereditary breast cancer. Grossly the cancer is soft and hemorrhagic. A rapid increase in size may occur secondary to necrosis and hemorrhage.

On PE, it is bulky and often positioned deep within the breast. Bilaterality is reported in 20% of cases.

It is characterized microscopically by:

a) a dense lymphoreticular infiltrate composed predominantly of lymphocytes.
b) large pleomorphic nuclei that are poorly differentiated and show active mitosis
c) a sheet-like growth pattern with minimal or absent ductal or alveolar differentiation.

50% are associated with DCIS, which characteristically is present at the periphery of the cancer, and <10% demonstrate hormone receptors.

There is an intense lymphocytic response, hence benign or hyperplastic enlargement of the LN of axilla may contibute to erroneous clinical staging.

Patients have a better 5-year survival rate than those with NST or invasive lobular carcinoma.

How well did you know this?
1
Not at all
2
3
4
5
Perfectly
74
Q

Discuss mucinous carcinoma (colloid carcinoma).

A

Mucinous/colloid caricnoma accounts for 2% of all invasive breast cancers and typically presents in the older population as a bulky tumor. This is defined by extracellular pools of mucin, which surround aggregates of low-grade cancer cells.

Cut surface shows a glistening and gelatinous quality.

Fibrosis is variable, and when abundant it imparts a firm consistency to the cancer. Over 90% of mucinous carcinomas display hormone receptors.

Lymph node mets occur in 33% of cases, and 5 and 10 year survival rates are 73% and 59%, respectively.

Because of the mucinous component, cancer cells may not be evident in all microscopic sections, hence analysis of multiple sections is essential to confirm the diagnosis.

How well did you know this?
1
Not at all
2
3
4
5
Perfectly
75
Q

Discuss papillary carcinoma of the breast.

A

Papillary carcinoma of the breast is a special type cancer that accounts for 2% of all invasive breast cancers. It generally presents in the seventh decade of life, and occurs in nonwhite women.

These are small (<3cm) and are defined by papillae with fibrovascular stalks and multilayered epithelium. 87% express estrogen receptor.

They have frequent axillary mets and similar 5-10year survival rates to mucinous and tubular carcinoma.

How well did you know this?
1
Not at all
2
3
4
5
Perfectly
76
Q

Discuss tubular carcinoma of the breast.

A

Tubular carcinoma of the breast is another special type breast cancer and accounts for 2% of all invasive breast cancers. It is reported in as many as 20% of women whose cancers are diagnosed by mammographic screening and usually is diagnosed in the perimenopausal or early menopausal periods.

94% of tubular cancers express estrogen receptor.

10% of women with tubular carcinoma or invasive cribriform carcinoma will develop axillary lymph node metastases. However the presence of metastatic disease in one or two axillary lymph nodes does not adversely affect survival.

Distant metastases are rare in tubular carcinoma and invasive cribriform carcinoma. Long term survival approaches 100%.

How well did you know this?
1
Not at all
2
3
4
5
Perfectly
77
Q

Discuss invasive lobular carcinoma.

A

Invasive lobular carcinoma accounts for 10% of breast cancers. The histopathologic features include small cells with rounded nuclei, inconspicuous nucleoli, and scant cytoplasm. Special stains may confirm the presence of intracytoplasmic mucin, which may displace the nucleus (signet-ring cell carcinoma).

It is often multifocal, multicentric, and bilateral. Because of its insidious growth pattern and subtle mammographic features, invasive lobular cancers express estrogen receptor.

How well did you know this?
1
Not at all
2
3
4
5
Perfectly
78
Q

What is the difference between the craniocaudal (CC) view and mediolateral-oblique (MLO) view in a mammogram?

A

The MLO view images the greatest volume of breast tissue, including the upper outer quadrant and the axillary tail of Spence.

Compared with the MLO view, the CC view provides better visualization of the medial aspect of the breast and permits greater breast compression.

Other views include the 90deg lateral and spot compression views. The 90deg lateral view is used along with the CC view to triangulate the exact location of an abnormality. Spot compression may be done in any projection by using a small compression device, which is placed directly over a mammographic abnormality that is obscured by overlying tissues.

Mammography is also used to guide interventional procedures including needle localization and needle biopsy.

How well did you know this?
1
Not at all
2
3
4
5
Perfectly
79
Q

What mammographic features suggest a diagnosis of breast cancer?

A

1) A solid mass with/without stellate features
2) Asymmetric thickening of breast tissues
3) Clustered microcalcifications
4) Fine, stippled calcium in and around a suspicious lesion (50% of nonpalpable cancers)

How well did you know this?
1
Not at all
2
3
4
5
Perfectly
80
Q

NCCN Guidelines of screening breast cancer?

A

Normal-risk women ≥20 years old should have a breast exam at least every 3 years.

Starting at age 40 years, breast examinations should be performed yearly, and a yearly mammogram should be taken.

Screening mammography in women ≥50 years old has been noted to reduce breast cancer mortality by 20-25%.

How well did you know this?
1
Not at all
2
3
4
5
Perfectly
81
Q

What is the primary indication for ductography?

A

Nipple discharge, particularly when the fluid contains blood.

In ductography, cancers may appear as irregular masses or multiple intraluminal filling defects.

How well did you know this?
1
Not at all
2
3
4
5
Perfectly
82
Q

What is the role of ultrasonography in breast cancer?

A

Ultrasonography is an important method of resolving equivocal mammographic findings, defining cystic masses, and demonstrating the echogenic qualities of specific solid abnormalities.

On ultrasound, breast cysts are well-circumscribed, with smooth margins and an echo-free center.

Benign breast masses show smooth contours, round or oval shapes, weak internal echoes, and well-defined anterior and posterior margins.

Breast cancer characteristically has irregular walls, but may have smooth margins with acoustic enhancement.

Ultrasonography is used to guide fine-needle aspiration biopsy, CNB, and needle localization of breast lesions.

It has highly reproducible findings, and a high patient acceptance rate, but it does not reliably detect lesions ≤1cm in diameter.

It can also be used to image the regional lymph nodes in patients with breast cancer. (Sensitivity of 35-82%, and specificity of 73-97%).

How well did you know this?
1
Not at all
2
3
4
5
Perfectly
83
Q

What are the features of a lymph node with cancer?

A
Cortical thickening
Change in shape of node to more circular appearance
>10mm in size
Absence of a fatty hilum
Hypoechoic internal echoes
How well did you know this?
1
Not at all
2
3
4
5
Perfectly
84
Q

What is the role of MRI in breast cancer?

A

There is current interest in the use of MRI to screen breasts of high-risk women and women with a newly diagnosed breast cancer (MRI of the contralateral breast in women with a known breast CA has shown a contralateral cancer in 5.7% of these women).

Some scenarios where MRI may be useful include: evaluation of a patient with nodal mets from breast CA without an identifiable primary, to assess response to therapy in the setting of neoadjuvant systemic treatment, to select patients for partial breast irradiation techniques, and evaluation of treated breast for tumor recurrence.

From NCCN:
1) Useful in finding a primary breast lesion for patients with positive malignant axillary nodes, but negative clinical/imaging evidence of a primary tumor.

2) More accurate than mammogram in assessing extent of primary tumor in young women (dense breast tissue) and in diagnosis of invasive lobular carcinoma (>90% sensitivity).
3) Early detection of cancer in patients with BRCA mutation

How well did you know this?
1
Not at all
2
3
4
5
Perfectly
85
Q

What is the only oncogene that acts by inhibiting apoptosis rather than by directly increasing cellular proliferation?

A

Bcl-2

How well did you know this?
1
Not at all
2
3
4
5
Perfectly
86
Q

What is the Oncotype DX?

A

The Oncotype DX is a 21-gene RT-PCR-based assay approved for use in newly diagnosed patients with node-negative, ER-positive breast cancer.

A recurrence score is generated, and those patients with high recurrence scores are likely to benefit from chemotherapy.

Those with low recurrence scores benefit most from endocrine therapy and may not required chemotherapy.

The Trial Assessing Individualized Options for Treatment for breast cancer (TAILORx), designed to prospectively validate the use of 21-gene expression assay, have shown that patients with low recurrence score (0-10) have a low rate of local-regional and distant recurrence (98.7%) and very good overall survival at 5 years (98%) with endocrine therapy alone.

Retrospective analysis has also shown that the 21-gene recurrence score can be use in postmenopausal patients with ER-positive tumors and 1-3 involved axillary lymph nodes to predict the benefit of chemotherapy in addition to endocrine therapy.

How well did you know this?
1
Not at all
2
3
4
5
Perfectly
87
Q

What is the MammaPrint assay?

A

The MammaPrint assay uses a 70-gene expression profile to assess the risk of distant metastasis.

This is FDA approved for use in stage 1 or stage 2 node negative, ER-positive, or ER-negative breast cancers to identify patients with high or low risk of recurrence.

The prospective RASTER study reported that patients classified as low risk based on MammaPrint had a 97% distant recurrence-free interval at five years.

Results of the prospective MINDACT (MicroarrayInNode negative and 1-3 positive lymph node Disease may Avoid ChemoTherapy) trial were recently reported. The study was designed to assess whether the 70-gene expression assay would help avoid chemotherapy in patients considered clinically high risk but categorized as low genomic risk based on the assay. A 5-year rate of distant metastasis-free survival of >92% was identified as the cutoff for the benefit of chemotherapy.

How well did you know this?
1
Not at all
2
3
4
5
Perfectly
88
Q

Management of In Situ Breast Cancer (Stage 0) - LCIS?

A

Stage 0
LCIS (marker for increased risk, 9-fold increased risk for CA)
Goal: Prevent or detect at an early stage the invasive cancer that subsequently develops in 25-35% of women
DX:
1) History and PE
2) Diagnostic bilateral mammography
3) Observation with interval history and PE every 6-12months
4) Annual mammogram

TX:
Advise regarding risk for Breast CA:
- Chemoprevention
- Bilateral total mastectomy
- Consider tamoxifen

There is NO benefit to excising LCIS because the disease diffusely involves both breasts in many cases and the risk of developing cancer is equal for both breasts.

How well did you know this?
1
Not at all
2
3
4
5
Perfectly
89
Q

Management of In Situ Breast Cancer (Stage 0) - DCIS?

A

Stage 0
DCIS
(10-fold increased risk for CA)

DX:

1) History & PE
2) Diagnostic bilateral mammography
3) ER status
4) MRI as indicated

TX:
With evidence of extensive disease (>4cm or in >1 quadrant)
1) Mastectomy
2) Adjuvant tamoxifen therapy (if ER positive)

Limited disease

1) Lumpectomy
2) Radiation therapy
3) Adjuvant tamoxifen therapy (if ER positive)

Nonpalpable DCIS

1) Needle-localization/ image-guided techniques
2) Specimen mammography is performed to ensure that all visible evidence of cancer is excised.
3) Adjuvant tamoxifen therapy (if ER positive)

The gold standard against which breast conservation therapy for DCIS is evaluated is mastectomy. Women treated with mastectomy have local recurrence and mortality rates of <2%.

How well did you know this?
1
Not at all
2
3
4
5
Perfectly
90
Q

What is the ECOG 5194?

A

The Eastern Cooperative Oncology group (ECOG) initiated a prospective registry trial (ECOG 5194) to identify those patients who could safely undergo breast-conserving surgery without radiation.

Eligible patients were those with low or intermediate grade DCIS measuring 2.5cm or less, who had negative margins of at least 3mm and those with high grade DCIS who had tumors measuring 1cm or less with a negative margin of at least 3mm.

At a median follow up of 6.2 years, patients with low or intermediate grade DCIS had an in-breast recurrence rate of 6.1% while those with high-grade DCIS had a recurrence rate of 15.3%.

Approximately 4% of patients developed a contralateral breast cancer during follow up in both low/intermediate and high-grade groups.

This study identified an acceptable recurrence rate for those patients with low or intermediate grade DCIS treated with excision alone with a margin of at least 3mm. In contrast, patients with high-grade DCIS had an unacceptable high local recurrence rate.

How well did you know this?
1
Not at all
2
3
4
5
Perfectly
91
Q

What is the role of axillary staging in patients with DCIS?

A

The role of axillary staging in patients with DCIS is limited. One consideration is for patients undergoing mastectomy.

Since most lesions are currently diagnosed with needle core biopsy, there is about a 20% incidence of invasive breast cancer on final pathologic assessment of the primary tumor.

Since it is not feasible to perform sentinel node dissection after mastectomy, most surgeons will recommend the use of sentinel node dissection at the time of mastectomy for DCIS.

How well did you know this?
1
Not at all
2
3
4
5
Perfectly
92
Q

Management of Locoregional Invasive Breast Cancer (Stage I, IIA, IIB, IIIA*— T3N1-2M0)

A

DX

1) History & PE
2) Diagnostic bilateral mammogram
3) Breast UTZ as necessary
4) ER-PR status determination
5) HER2 status determination
6) Pathology review

TX:
1) Mastectomy with Axillary Staging (SLN dissection for clinically N0 patients– ie, Modified Radical Mastectomy)

or

2) Breast Conserving Surgery (Lumpectomy or Wide Excision + RT or + SLNB)
- Not advised in women who are known BRCA mutation carriers, due to high lifetime risk for development of additional breast cancers.

+

Reconstruction

If post-mastectomy RT is needed, a tissue expander can be placed at the time of mastectomy to save the shape of the breast and reduce the amount of skin replacement needed. The expander can be deflated at the initiation of radiation therapy to allow for irradiation of the chest wall and regional nodal basins. Removal of the tissue expander and definitive reconstruction, usually with autologous tissue, can proceed 6 months to 1 year after completion of RT.

How well did you know this?
1
Not at all
2
3
4
5
Perfectly
93
Q

Relative contraindications to breast conservation therapy?

A

1) Prior radiation therapy to the breast or chest wall
2) Persistently positive surgical margins after re-excision
3) Multi-centric disease
4) Scleroderma or lupus erythematosus

How well did you know this?
1
Not at all
2
3
4
5
Perfectly
94
Q

When should adjuvant chemotherapy be considered for breast cancer, based on the ASCO guidelines?

A

Adjuvant chemotherapy should be considered for patients with:

1) Positive lymph nodes
2) ER-negative disease
3) HER2-positive disease
4) Adjuvant! Online mortality >10%
5) Grade 3 node-negative tumors >5mm
6) Triple negative tumors
7) Lymphovascular invasion
8) Estimated distant relapse risk of >15% at 10 years based on the 21 gene recurrence score assay

NCCN:

1) >2cm tumor
2) Poor histologic & nuclear grade
3) Negative hormone receptors
4) High proliferative fraction
5) HER2 expression
6) All node positive tumors

How well did you know this?
1
Not at all
2
3
4
5
Perfectly
95
Q

When should adjuvant endocrine therapy be considered in patients with breast cancer?

A

Adjuvant endocrine therapy is considered for:
Hormone (+) cancers
(Aromatase inhibitor if patient is postmenopausal)

HER2/neu status is determined for all patients with newly diagnosed invasive breast cancer and when positive, should be used to guide systemic therapy recommendations.

How well did you know this?
1
Not at all
2
3
4
5
Perfectly
96
Q

Indications for total mastectomy in DCIS?

A

1) Diffuse, suspicious mammographic calcifications
2) Inability to obtain clear margins on wide excision
3) Poor cosmetic result
4) Contraindications to RT
5) Patient’s choice

How well did you know this?
1
Not at all
2
3
4
5
Perfectly
97
Q

Indications for RT post-mastectomy?

A

1) ≥ 4 positive nodes
2) >5cm tumors (T3)
3) Very aggressive histology (diffuse vascular invasion)
4) Extranodal extension
5) Positive surgical margins
6) Inflammatory breast cancer
7) Involvement of skin, pectoral fascia, or skeletal muscle

How well did you know this?
1
Not at all
2
3
4
5
Perfectly
98
Q

Management of Atypical Ductal/Lobular Hyperplasia?

A

Atypical ductal/lobular hyperplasia has a 4-fold increased risk for malignancy.

TX:

1) Wire-localized excisional biopsy
2) Start Tamoxifen 20mg/day x 5 years

How well did you know this?
1
Not at all
2
3
4
5
Perfectly
99
Q

Components of the Gail Model?

A

1) Age
2) Age at Menarche
3) Age at first live birth
4) Family history
5) Number of breast biopsies

How well did you know this?
1
Not at all
2
3
4
5
Perfectly
100
Q

Five-year survival rates for breast cancer, if treated?

A
Stage I: 95-100%
Stage II: 80-90%
Stage III: 50-70%
Stage IV: 1-20%
Tis: 98-100%
How well did you know this?
1
Not at all
2
3
4
5
Perfectly
101
Q

Candidates for skin-sparing mastectomy?

A

Skin-sparing mastectomy is the excision of nipple/areola complex + skin around biopsy site, + skin within 1-2cm of tumor margin. 5-10% of skin is removed. May include SLNB (if positive–> ALND).

Candidates:

1) Multicentric disease
2) Invasive CA with extensive intraductal component
3) T2 tumors with difficult to interpret mammograms
4) Central tumors requiring removal of nipple-areola complex

How well did you know this?
1
Not at all
2
3
4
5
Perfectly
102
Q

Candidates for nipple-sparing mastectomy?

A

1) No clinical involvement of NAC
2) <5cm tumor
3) Tumor-nipple distance ≥2cm

How well did you know this?
1
Not at all
2
3
4
5
Perfectly
103
Q

Indications for level I & II axillary dissection?

A

1) Clinically positive nodes at diagnosis, confirmed by FNAB or CNB
2) Sentinel nodes not identified

In the absence of gross disease in level II nodes, dissection should include tissue inferior to the axillary vein from the lat dorsi, laterally to the medial border of the pex minor.

How well did you know this?
1
Not at all
2
3
4
5
Perfectly
104
Q

Indications for preoperative systemic therapy?

A

1) Locally advanced or inoperable breast tumors (inflammatory breast CA)
2) N2-N3 disease
3) T4 tumors
4) Operable breast CA who are clear candidates for adjuvant chemotherapy, and desire BCT
5) Breast cancer subtype with high likelihood of response

How well did you know this?
1
Not at all
2
3
4
5
Perfectly
105
Q

Options for adjuvant endocrine therapy for POSTmenopausal women (early breast CA)?

A

1) Aromatase inhibitor x 5 year
2) Tamoxifen x 2-3 years followed by:
- AI x 5 years or
- Tamoxifen x 4.5-6 years followed by AI x 5 years or
- Tamoxifen x 10 years

How well did you know this?
1
Not at all
2
3
4
5
Perfectly
106
Q

Mechanism of action of Aromatase Inhibitors?

A

AIs block conversion of adrenally synthesized androgens to estrogen (final step to conversion to active form of the hormone).

How well did you know this?
1
Not at all
2
3
4
5
Perfectly
107
Q

Options for adjuvant endocrine therapy for PREmenopausal women (early breast CA)?

A

1) Tamoxifen x 5 years, with or without suppression (adjuvant) or
2) Ovarian suppression + AI x 5 years

How well did you know this?
1
Not at all
2
3
4
5
Perfectly
108
Q

Mechanism of action and dose of Tamoxifen?

A

Tamoxifen 20mg/tab OD

Tamoxifen is a competitive ER antagonist.

Side effects: Hot flushes, vaginal dryness/discharge, thromboembolic events, endometrial CA

How well did you know this?
1
Not at all
2
3
4
5
Perfectly
109
Q

Management of advanced locoregional breast cancer (Stage IIIA or IIIB)?

A

DX:

1) History & PE
2) CBC
3) Liver FT + Alk phos
4) Chest imaging
5) Pathology review
6) Pre-chemo ER/PR status, HER2 status
7) Diagnostic bilateral mammogram
8) UTZ as necessary

TX:

1) Neoadjuvant chemotherapy (Anthracycline based)
2) Modified Radical Mastectomy
3) Adjuvant RT
4) Endocrine/Trastuzumab therapy as warranted.

If no response to adjuvant tx:

1) Additional chemotx or
2) Preoperative RT

How well did you know this?
1
Not at all
2
3
4
5
Perfectly
110
Q

Surveillance/follow up for Stage I-III breast cancer?

A

1) History & PE
2) Mammogram annually
3) If on Tamoxifen: annual gynecologic assessment
4) If on AI: Bone health + bone mineral density tests
5) Active lifestyle, healthy diet, maintain IBW (20-25 BMI)

How well did you know this?
1
Not at all
2
3
4
5
Perfectly
111
Q

Management of metastatic or recurrent breast cancer (Stage IV)?

A

DX:

1) History & PE
2) CBC
3) LFTs
4) Chest CT
5) Bone Scan
6) X-ray of painful weight bearing bones
7) Biopsy documentation of mets if possible
8) ER-PR, HER-2 status

TX:

1) Endocrine treatment
2) Systemic chemotherapy
3) Bisphosphonates (Bone mets)

How well did you know this?
1
Not at all
2
3
4
5
Perfectly
112
Q

Management of Phyllodes Tumor?

A

DX:

1) History & PE
2) UTZ
3) Mammogram if ≥30 years old
4) Core needle biopsy

TX:

1) Wide excision
2) Mastectomy for larger lesions

Recurrence:
Positive Mets: Follow soft tissue sarcoma guidelines
Negative Mets: Re-excision with wide margins ≥1cm
Negative axillary staging, consider postop RT

How well did you know this?
1
Not at all
2
3
4
5
Perfectly
113
Q

Management of Paget’s disease of the breast?

A

Rare manifestation of cancer, neoplastic cells in the epidermis of the nipple-areolar complex. Presents with eczema, bleeding, ulceration, itching. Associated cancer in the breast elsewhere in up to 80-90% of cases.

DX:
1) History & PE and Imaging (+) for breast mass:
Core needle biopsy of lesion + full thickness skin

2) History & PE and Imaging (-) for breast mass:
Full-thickness biopsy

3) Biopsies (-) for CA, but (+) for Paget’s:
Breast MRI and tissue sampling

TX:
1) (-) Mass or imaging abnormality:
Central lumpectomy (NAC + margin of underlying breast tissue)

2) Mass in other part of the breast:
Removal of NAC + negative margins + BCS for peripheral CA

3) Mastectomy (if DCIS)

How well did you know this?
1
Not at all
2
3
4
5
Perfectly
114
Q

Management of breast cancer during pregnancy?

A

Breast cancer during pregnancy is often ALN-positive, larger primary tumor size, and often with poorly differentiated tumors.

DX:

1) History & PE of breast and lymph node status
2) Mammogram with shielding
3) Ultrasound of breast and LN
4) FNAB or CNB (preferred) with ER-PR/HER2 analysis
5) CXR with shielding, include FTs and renal FTs, CBC (T1-T2)
6) Add liver UTZ and thoracolumbar MRI without contrast (N+ or T3)

TX:

1) BCS (Lumpectomy/wide excision + RT; RT can be delayed to postpartum)
2) MRM
3) Chemotherapy: For 2nd and 3rd trimesters ONLY with fetal monitoring prior to each chemo (BUT NOT >35 weeks AOG to prevent bleeding)

CONTRAINDICATIONS

  • Trastuzumab, RT, AI, Tamoxifen: ONLY POSTPARTUM
  • SLNB: Contraindicated for <30 weeks AOG
  • Chemotherapy: Contraindicated for 1st trimester and after 35 weeks AOG.
How well did you know this?
1
Not at all
2
3
4
5
Perfectly
115
Q

Management of Inflammatory Breast Cancer?

A

IBC presents as erythema and dermal edema of ≥1/3 of skin of breast. It is often ER-PR/HER2 +. Considered as IIIB, IIIC, or Stage IV (T4d), is comprises <3% of breast CA. It is caused by blockage of lymph by tumor emboli.

DX:

1) History & PE
2) CBC + Plt
3) Pathology review (ER-PR/HER2)
4) Diagnostic bilateral mammogram
5) UTZ & MRI as necessary
6) Mets workup: LFTs, Bone scan, Chest/Abd/Pelvic CT, PET/CT for equivocal findings

TX:

1) Neoadjuvant chemotherapy
2) Mastectomy (with I/II ALND, ie MRM)
3) RT
4) HER2 therapy if (+)

How well did you know this?
1
Not at all
2
3
4
5
Perfectly
116
Q

Contraindications to Sentinel Lymph Node dissection?

A

SLND is used to assess the regional LN in early breast cancer who are clinically node negative.

Contraindications to SLND:

1) Inflammatory breast CA
2) Palpable axillary lymph nodes
3) Biopsy-proven metastases
4) DCIS without mastectomy
5) Prior axillary surgery

Tumors in lateral breast are likely to have a false negative SLN.

How well did you know this?
1
Not at all
2
3
4
5
Perfectly
117
Q

BIRADS Scoring System?

A

0: Incomplete
1: Normal
2: Benign
3: Probably Benign
4: Possibly Malignant (Suspicious)
5: Malignant (Highly suspicious)
6: Malignant Tissue

How well did you know this?
1
Not at all
2
3
4
5
Perfectly
118
Q

Discuss classifications for endocrine status.

A

The binding of estrogen to its receptor induces PR expression. The presence of both receptors in a tumor indicates an 80% chance of a favorable response to endocrine therapy.

LUMINAL A (ER+ PR+ HER2-)

  • low-grade, 40% of all breast CA
  • good prognosis
  • TX:
    1) Premenopause: SERMs (Tamoxifen)
    2) Postmenopause: AIs (Anastrozole, Exemestane, Letrozole)

LUMINAL B (ER+ PR+/- HER2+/-)

  • higher grade, 20%
  • poorer prognosis
  • TX:
    1) Tamoxifen (decreased response)
    2) Chemotherapy (increased response to chemo)

HER2-ENRICHED (ER- PR- HER2+)

  • High-grade, often node positive
  • p53 mutations
  • 10-15% of all breast CA
  • TX: Trastuzumab, Pertuzumab, Lapatinib

BASAL-LIKE (ER- PR- HER2-)

  • Triple negative
  • High proliferation, BRCA dysfunction
  • 15-20%
  • Poor prognosis
  • TX: Chemotherapy
How well did you know this?
1
Not at all
2
3
4
5
Perfectly
119
Q

What is the 21-gene assay?

A

Oncotype Dx (21-gene assay) is the only assay clinically validated, using a 21-gene recurrence score (RS) for:

1) Predicting the benefit of adding adjuvant chemotherapy to adjuvant endocrine therapy, to further reduce the risk of recurrence in women with HR-positive, HER2-negative and node-negative breast cancer.
2) Determining the prognosis in women with HR-positive, HER2-negative tumors treated with endocrine therapy alone by predicting locoregional and distant recurrence.

How well did you know this?
1
Not at all
2
3
4
5
Perfectly
120
Q

What is the 70-gene assay?

A

MammaPrint (70-gene assay) can identify a subset of patients who have a low likelihood of distant recurrence despite a high risk clinical features (based on tumor size, grade and nodal status).

How well did you know this?
1
Not at all
2
3
4
5
Perfectly
121
Q

What is Adjuvant! Online?

A

Adjuvant! Online is a validated computer-based model available to estimate 10-year DFS and OS that incorporates all of the above prognostic factors except for HER2 tumor status.

How well did you know this?
1
Not at all
2
3
4
5
Perfectly
122
Q

What are the RECIST Guidelines?

A

Response Evaluation Criteria in Solid Tumors (RECIST) was introduced in 2000 to standardize and simplify tumor response criteria.

V 1.1 (2020)

TARGET LESIONS
CR: Disappearance of all target lesions and reduction in the short axis measurement of all pathologic lymph nodes to ≤10 mm

PR: ≥30% decrease in the sum of the longest diameter of the target lesions compared with baseline

PD: ≥20% increase of at least 5 mm in the sum of the longest diameter of the target lesions compared with the smallest sum of the longest diameter recorded

or

The appearance of new lesions, including those detected by FDG-PET

SD: Neither PR nor PD

NON-TARGET LESIONS
CR: Disappearance of all non-target lesions and normalization of tumor marker levels

IR, SD: Persistence of 1 or more non-target lesions and/or the maintenance of tumor marker levels above normal limits

PD: The appearance of 1 or more new lesions or unequivocal progression

If patient has measurable disease, an increase in the overall level or substantial worsening in non-target lesions, such that tumor burden has increased, even if there is SD or PR in target lesions

If no measurable disease, an increase in the overall tumor burden comparable in magnitude with the increase that would be required to declare PD in measurable disease (eg, an increase in pleural effusions from trace to large, or an increase in lymphangitic disease from localized to widespread)

How well did you know this?
1
Not at all
2
3
4
5
Perfectly
123
Q

Appropriate candidates for initial endocrine therapy in stage IV breast CA patients?

A

Women with hormone receptor positive cancers with no immediate life-threatening disease (or visceral crisis) (ie, only bone or soft tissue mets, and those with limited visceral mets).

How well did you know this?
1
Not at all
2
3
4
5
Perfectly
124
Q

Indications for systemic chemotherapy in stage IV breast CA patients?

A

Women with hormone receptor-negative cancers, visceral crisis, and hormone-refractory metastases.

How well did you know this?
1
Not at all
2
3
4
5
Perfectly
125
Q

Agents given to women with bone metastases in stage IV breast CA?

A

Bisphosphonates

Anti-RANKL (Receptor Activator of Nuclear Factor Kappa-B Ligand) agent, Denosumab

How well did you know this?
1
Not at all
2
3
4
5
Perfectly
126
Q

How do you manage local-regional recurrence?

A

Women post-mastectomy:

  • Surgical resection of local-regional recurrence and reconstruction
  • Consider chemotherapy and antiestrogen therapy
  • Adjuvant RT if no previous RT

Women post-lumpectomy/BCS:

  • Mastectomy
  • Reconstruction
  • Consider chemo and antiestrogen therapy depending on hormone status and HER2 status of the tumor.
How well did you know this?
1
Not at all
2
3
4
5
Perfectly
127
Q

When is excisional biopsy used?

A

When needle biopsy results are discordant with imaging findings or PE.

How well did you know this?
1
Not at all
2
3
4
5
Perfectly
128
Q

What type of incision is recommended for excision biopsies in the lower half of the breast?

A

Radial incisions

How well did you know this?
1
Not at all
2
3
4
5
Perfectly
129
Q

Why are radial incisions not recommended for the upper half of the breast?

A

Because of possible scar contracture resulting in displacement of the ipsilateral nipple-areola complex.

Similarly, curvilinear incisions in the lower half of the breast may displace the NAC downward.

How well did you know this?
1
Not at all
2
3
4
5
Perfectly
130
Q

How many lymph nodes are needed to stage the axilla in SLN biopsy?

A

Studies have demonstrated that 98% of all positive SLNs will be recovered with the removal of 4 SLNs. Therefore, it is not necessary to remove greater than 4 SLNs for accurate staging of the axilla.

How well did you know this?
1
Not at all
2
3
4
5
Perfectly
131
Q

What is a skin-sparing mastectomy?

A

A skin-sparing mastectomy removes all breast tissue, NAC, and scars for prior biopsy procedures.

There is a recurrence rate of <6-8%, comparable to the long-term recurrence rates of standard mastectomy, when skin-sparing mastectomy is used for patients with Tis to T3 cancers.

How well did you know this?
1
Not at all
2
3
4
5
Perfectly
132
Q

What is a total (simple) mastectomy?

A

A total (simple) mastectomy without skin sparing removes all breast tissue, the NAC, and skin.

How well did you know this?
1
Not at all
2
3
4
5
Perfectly
133
Q

What is an extended simple mastectomy?

A

An extended simple mastectomy removes all breast tissue, the NAC, skin, and level I axillary lymph nodes.

How well did you know this?
1
Not at all
2
3
4
5
Perfectly
134
Q

What is a modified radical “Patey” mastectomy?

A

An MRM removes all breast tissue, the NAC, skin, and levels I, II, and III axillary lymph nodes.

The pectoralis minor divided and removed by Patey may be simply divided, giving improved access to level III nodes, then left in situ, or occasionally the axillary clearance can be performed without dividing pectoralis minor.

How well did you know this?
1
Not at all
2
3
4
5
Perfectly
135
Q

What is the Halsted radical mastectomy?

A

The Halsted radical mastectomy removes all breast tissue and skin, the NAC, the pectoralis major and minor muscles, and levels I, II, and III axillary lymph nodes.

The use of systemic chemotherapy and hormonal therapy as well as adjuvant radiation therapy for breast cancer have nearly eliminated the need for radical mastectomy.

How well did you know this?
1
Not at all
2
3
4
5
Perfectly
136
Q

What is nipple-areolar sparing mastectomy?

A

Nipple-areolar sparing mastectomy has been popularized over the last decade especially for risk-reducing mastectomy in high-risk women.

Factors for eligibility:

1) Tumor located >2 to 3cm from the border of the areola
2) Smaller breast size
3) Minimal ptosis
4) No prior breast surgeries with periareolar incisions
5) BMI <40kg/m^2
6) No active tobacco use
7) No prior breast irradiation
8) No evidence of collagen vascular disease

How well did you know this?
1
Not at all
2
3
4
5
Perfectly
137
Q

Anatomic boundaries of the modified radical mastectomy?

A

1) Anterior margin of the latissimus dorsi laterally
2) Midline of the sternum medially
3) Subclavius muscle superiorly
4) 2-3cm inferior to the inframammary fold inferiorly

How well did you know this?
1
Not at all
2
3
4
5
Perfectly
138
Q

Skin-flap thickness in MRM?

A

7-8mm inclusive of skin and telasubcutanea. May differ with body habitus.

How well did you know this?
1
Not at all
2
3
4
5
Perfectly
139
Q

Current recommendations for RT in Stage IIIA and IIIB breast cancer?

A

1) Adjuvant radiation therapy to the breast and supraclavicular lymph nodes after neoadjuvant chemotherapy, and segmental mastectomy with or without axillary node dissection
2) Adjuvant radiation therapy to the chest wall and supraclavicular lymph nodes after neoadjuvant chemotherapy and mastectomy with or without axillary lymph node dissection
3) Adjuvant radiation therapy to the chest wall and supraclavicular lymph nodes after segmental mastectomy or mastectomy with axillary lymph node dissection and adjuvant chemothearpy.q

How well did you know this?
1
Not at all
2
3
4
5
Perfectly
140
Q

Candidates for adjuvant chemotherapy?

A

1) Positive lymph nodes
2) HER2-positive disease
3) Adjuvant! Online greater mortality >10%
4) Grade 3 LN negative tumors >5mm
5) Triple-negative tumors
6) LV invasion
7) Estimated distant relapse risk of 15% at 10 years based on 21 gene recurrence score
8) Hormone receptor negative cancers >1cm

How well did you know this?
1
Not at all
2
3
4
5
Perfectly
141
Q

Adjuvant chemotherapy regimens for HER2-negative breast CA?

A

Preferred:

1) Dose dense AC –> Paclitaxel every 2 weeks
2) Dose dense AC –> Paclitaxel weekly
3) TC (T = docetaxel)

Others:

1) CMF
2) AC –> Docetaxel every 3 weeks
3) AC –> Paclitaxel weekly
4) TAC (T = docetaxel)

How well did you know this?
1
Not at all
2
3
4
5
Perfectly
142
Q

Adjuvant chemotherapy regimens for HER2-positive breast CA?

A

Preferred:
AC –> T + trastuzumab +/- pertuzumab (T = paclitaxel)
TCH (docetaxel, carboplatin, trastuzumab +/- pertuzumab

How well did you know this?
1
Not at all
2
3
4
5
Perfectly
143
Q

AJCC T Staging for Breast Cancer?

A

TX: Primary tumor cannot be assessed
T0: No evidence of primary tumor
Tis (DCIS): Ductal carcinoma in situ (DCIS)

Tis (Paget): Paget disease of the nipple not associated with invasive carcinoma and/or carcinoma in situ (DCIS) in the underlying breast parenchyma. Carcinomas in the breast parenchyma associated with Paget disease are categorized based on the size and characteristics of the parenchymal disease, although the presence of Paget should still be noted.

T1: Tumor ≤20mm in greatest dimension
T1mi: Tumor ≤1mm
T1a: Tumor >1mm but ≤5mm
T1b: Tumor >5mm but ≤10mm
T1c: Tumor >10mm but ≤20mm

T2: Tumor >20mm but ≤50mm in greatest dimension
T3: Tumor >50mm in greatest dimension

T4: Tumor of any size with direct extension to the chest wall and/or to the skin (ulceration or macroscopic nodules); invasion of dermis alone does not qualify as T4
T4a: Extension to chest wall, invasion/adherence to pex in absence of chest wall structures does NOT qualify as T4
T4b: Ulceration and/or ipsilateral macroscopic satellite nodules and/or edema (inc peau d’orange) of the skin that does not meet criteria for inflammatory carcinoma
T4c: Both T4a and T4b present
T4d: Inflammatory carcinoma

How well did you know this?
1
Not at all
2
3
4
5
Perfectly
144
Q

AJCC cN Staging for Breast Cancer?

A

cNX: Regional lymph nodes cannot be assessed (ie, previously removed)
cN0: No regional lymph node mets (by imaging or clinical exam)

cN1: Metastases to movable ipsilateral level I, II axillary lymph nodes
cN1mi: Micrometastases

cN2: Mets in ipsilateral level I, II axillary LNs that are clinically fixed or matted; or in ipsilateral internal mammary nodes in the absence of axillary LN mets
cN2a: Mets in ipsilateral level I, II axillary LN fixed to one another (matted) or to other structures
cN2b: Mets only in ipsilateral internal mammary nodes in the absence of axillary LN mets

cN3: Mets in ipsilateral infraclavicular (level III axillary) LNs with or without level I, II axillary LN involvement; or in ipsilateral internal mammary LN with level I, II axillary LN mets; or mets in ipsilateral supraclavicular LN with or without axillary or internal mammary lymph node involvement

cN3a: Mets in ipsilateral infraclavicular lN
cN3b: Mets in ipsilateral internal mammary LNs and axillary LNs
cN3c: Mets in ipsilateral supraclavicular LNs

How well did you know this?
1
Not at all
2
3
4
5
Perfectly
145
Q

AJCC M Staging for Breast Cancer?

A

M0: No clinical or radiographic evidence of distance metastases
cMO(i+): No clinical or radiographic evidence of distant mets in the presence of tumor cells or deposits not larger than 0.2mm detected microscopically or by molecular techniques in circulating blood, bone marrow, or other nonregional nodal tissue in a patient without symptoms or signs of mets.

cM1: Distant mets detected by clinical and radiographic means

pM1: Any histologically proven mets in distant organs, or if in non-regional nodes, mets greater than 0.2mm.

How well did you know this?
1
Not at all
2
3
4
5
Perfectly
146
Q

AJCC Anatomic Staging for Breast Cancer?

A

Stage 0: Tis N0 M0
Stage IA: T1 N0 M0
Stage IB: T0-1 N1mi M0

Stage IIA: T0-1 N1 M0
T2 N0 M0

Stage IIB: T2 N1 M0
T3 N0 M0

Stage IIIA: T0-2 N2 M0
T3 N1-2 M0

Stage IIIB: T4 N0-2 M0
Stage IIIC: Any T N3 M0
Stage IV: Any T Any N M1

147
Q

AJCC Histologic Grade for Breast Cancer?

A
INVASIVE CANCER (Scarff-Bloom-Richardson Grading System, Nottingham Modification):
GX: Grade cannot be assessed

G1: Low combined histologic grade (favorable); SBR score of 3-5 points

G2: Intermediate combined histologic grade (moderately favorable); SBR score of 6-7 points

G3: High combined histologic grade (unfavorable); SBR score of 8-9 points

DUCTAL CARCINOMA IN-SITU
GX: Grade cannot be assessed
G1: Low nuclear grade
G2: Intermediate nuclear grade
G3: High nuclear grade
148
Q

Breast cancer tumors are HER2-positive if?

A

They are scored as 3+ by an IHC method defined as uniform membrane staining for HER2 in 10% or more of tumor cells, or demonstrate HER2 gene amplification by an ISH method.

149
Q

IHC are equivocal if?

A

Scored as IHC 2+ based on circumferential membrane staining that is incomplete and/or weak/moderate and within greater than 10% of the invasive tumor cells, or complete and circumferential membrane staining that is intense and within less than or equal to 10% of the invasive tumor cells.

150
Q

Because of overlapping toxicities, trastuzumab is NOT concurrently given with?

A

Anthracyclines (Doxorubicin)

151
Q

Patients with HER2-positive tumors benefit is trastuzumab is added to?

A

Taxane chemotherapy (Paclitaxel)

152
Q

When is nipple discharge suggestive of cancer?

A

When it is spontaneous, unilateral, localized to a single duct, present in ≥40yo, bloody, or associated with a mass.

Mammography, ultrasound, and possibly a ductogram are indicated.

Definitive diagnosis depends on excisional biopsy of the offending duct and any associated mass lesion.

Localization with a wire or seed is performed when there is an associated mass that lies >2.0-3.0cm from the nipple.

153
Q

When is nipple discharge suggestive of a benign condition?

A

When it is bilateral, multiductal in origin, occurs in women ≤39yo, or is milky or blue-green.

Prolactin-secreting pituitary adenomas are responsible for bilateral nipple discharge in <2% of cases.

If serum prolactin levels are repeatedly elevated, plain radiographs of the sella turcica are indicated, and thin section CT scan is required.

Optical nerve compression, visual field loss, and infertility are associated with large pituitary adenomas.

154
Q

What is the probability of harboring cancer in a woman with axillary lymph node mets?

A

A woman who presents with axillary lymph node mets consistent with a breast cancer metastasis has a 90% probability of harboring an occult breast cancer.

However, axillary lymphadenopathy is the initial presenting sign in only 1% of breast cancer patients.

The search for a primary cancer should include examination of the thyroid, breast, and pelvis, including rectum.

155
Q

Discuss male breast cancer.

A

<1% of all breast cancers occur in men. Male breast cancer is preceded by gynecomastia in 20% of men.

It is associated with radiation exposure, estrogen therapy, testicular feminizing syndromes, and Klinefelter’s syndrome (XXY).

Breast cancer is rarely seen in young males and has a peak incidence in the 6th decade of life.

DCIS makes up <15% of male breast cancer, whereas infiltrating ductal carcinoma makes up >85%.

Stage by stage, males have the same survival rate as women. Overall, men do worse because of the more advanced stage of their cancer (stage II, III or IV) at the time of diagnosis.

80% of male breast cancers are HR-positive, and adjuvant tamoxifen is considered.

156
Q

What is the incidence of clinical node positivity in inflammatory breast cancer?

A

> 75% of women with inflammatory breast cancer present with palpable axillary lymphadenopathy, and distant mets are also frequently present.

157
Q

What is the recommended imaging test for young patients who are at high risk in developing breast cancer?

a. Mammography
b. Breast Ultrasound
c. Breast MRI
d. Ductography

A

c. Breast MRI

158
Q

What would you recommend to a patient with a BIRADS 4 mammographic reading?

a. Routine screening mammography
b. Excision of the lesion
c. Core needle biopsy of the lesion
d. Additional imaging procedures

A

c. Core needle biopsy of the lesion

159
Q

Which of the following findings show a favorable prognosis for patients who underwent surgical treatment for breast cancer?

a. ER/PR (+), HER-2/neu (-)
b. ER/PR (+), HER-2/neu (+)
c. ER/PR (-), HER-2/neu (-)
d. ER/PR (-), HER-2/neu (+)

A

a. ER/PR (+), HER-2/neu (-)

160
Q

A 35-year old woman with no family history of breast cancer is seen for a 2 cm hard mass in the upper outer quadrant of her left breast. FNAB reveals carcinoma. She undergoes conservative breast surgery. The specimen confirms that the lesion is invasive duct carcinoma with 2 positive axillary nodes and negative for estrogen receptors. What would you recommend for the patient?

a. Radiation to the residual breast
b. Radiation plus chemotherapy
c. Completion total mastectomy
d. Radiation plus tamoxifen
e. Radiation, chemotherapy and tamoxifen

A

b. Radiation plus chemotherapy

161
Q

A 55-year old female underwent MRM for Stage II breast cancer. Histopath revealed a 2 cm mass, nuclear and histologic grade 2, negative 21/21 axillary lymph nodes; lines of resection negative for tumor. Receptor assay is negative for both estrogen and progesterone. Based on current evidence, which of the ff adjuvant treatment would be most beneficial?

a. None
b. Radiotherapy
c. Chemotherapy
d. Hormonal therapy

A

c. Chemotherapy

162
Q

Which of the ff pairs represent valid treatment options for a 3 cm focus of DCIS?

a. MRM or mastectomy or lumpectomy, axillary dissection and RT
b. Lumpectomy or lumpectomy and RT
c. Lumpectomy and RT or simple mastectomy
d. MRM or simple mastectomy
e. Lumpectomy, axillary dissection, and RT or lumpectomy and RT

A

c. Lumpectomy and RT or simple mastectomy

163
Q

Which of the following drugs can be considered for use as adjuvant hormone therapy for ductal carcinoma in situ?

a. Anastrazole
b. Fulvestrant
c. Goserelin
d. Tamoxifen

A

d. Tamoxifen

164
Q

Which of the following carries the highest risk of developing breast cancer?

a. proliferative disease without atypia
b. lobular carcinoma in situ
c. proliferative disease with atypia
d. nonproliferative disease

A

b. lobular carcinoma in situ

165
Q

A 57-year-old female comes to the clinic with complaints of recent onset unilateral breast swelling and redness. She had previously been seen three weeks ago for the same matter, however, the symptoms continued despite treatment with a full course of oral antibiotics for a presumed mastitis. You perform punch biopsies of thickened areas of skin as well as core biopsies of a palpable mass with associated lymphadenopathy. The results come back positive for invasive ductal cancer with dermal lymphatic invasion. Which of the following is the MOST appropriate course of treatment for this condition?

a. Modified radical mastectomy with adjuvant radiation
b. Breast conservation surgery with chemotherapy and hormonal therapy
c. Systemic chemotherapy and radiation therapy only
d. Neoadjuvant chemotherapy followed by surgery and radiation therapy

A

d. Neoadjuvant chemotherapy followed by surgery and radiation therapy

166
Q

A 35-year-old female presents to her primary care physician asking about screening for breast cancer. She is healthy and does not have any palpable breast masses. She has a family history of breast cancer in her grandmother that was diagnosed at 65 years old and in her mom that was diagnosed at 60 years old. What should be the next step in screening this woman?

a. Mammogram yearly starting now
b. Mammogram yearly starting at age 40
c. Mammogram followed by MRI now and yearly
d. Mammogram and BRCA testing now

A

b. Mammogram yearly starting at age 40

167
Q

The embryologic structure from which the breast forms:

A

Ectodermal thickenings, termed mammary ridges or milk lines

168
Q

Name the function of the following hormones:

A

Estrogen: branching differentiation and duct development in the breast Progesterone: lobular development of the breast

169
Q

Name the muscle the artery supplies:

A

Lateral thoracic artery: Serratus anterior muscle

Thoracodorsal artery: Latissimus dorsi

170
Q

Name the nerve that innervates the following muscles:

A

Serratus anterior muscle: Long thoracic nerve
Latissimus dorsi: Thoracodorsal nerve
Pectoralis minor: Medial pectoral nerve
Pectoralis major: Lateral and medial pectoral nerves

171
Q

Name the complication if the following nerves were injured:

A

Long thoracic nerve: Winged scapula

Thoracodorsal nerve: Weak arm adduction/pull-ups

172
Q

Name the arterial supply to the breast:

A

Branches derived from the intercostal arteries,
internal thoracic artery,
lateral thoracic artery, and
thoracoacromial artery

173
Q

The valveless venous plexus responsible for direct hematogenous spread of breast cancer to the spine:

A

Batson plexus

174
Q

Suspensory ligaments that divide the breast into segments:

A

Cooper ligaments

175
Q

Percentages of lymphatic drainage of the breast:

A

The axillary nodes: 97%

The internal mammary nodes: 1% to 2%

176
Q

Anatomic description for levels I, II, and III nodes in the breast:

A

Level I: Lateral to the pectoralis minor muscle
Level II: Beneath the pectoralis minor muscle
Level III: Medial to the pectoralis minor muscle

177
Q

What are Rotter nodes?

A

Nodes between the pectoralis minor and major muscles

178
Q

What are the boundaries of the axilla?

A

Superior: Axillary vein
Posterior: Long thoracic nerve
Lateral: Latissimus dorsi muscle
Medial: Pectoralis minor

179
Q

Nerves to be aware of in an axillary lymph node dissection (ALND):

A
Long thoracic nerve
Thoracodorsal nerve
Medial pectoral nerve
Lateral pectoral nerve 
Intercostobrachial nerve
180
Q

Potential complications of ALND:

A
Axillary vein thrombosis Infection
Nerve injury
Lymphedema
Lymphatic fibrosis
Lymphangiosarcoma
181
Q

Most likely cause of sudden, painful early postop swelling of the ipsilateral arm after an axillary dissection:

A

Axillary vein thrombosis

182
Q

Most likely cause of slow, painless, progressive swelling of the ipsilateral arm after an axillary disseciion:

A

Lymphatic fibrosis

183
Q

Most likely cause of hyperesthesia of the inner upper aspect of the ipsilateral arm after an axillary dissection:

A

Injury to the second intercostobrachial cutaneous nerve

184
Q

Incidence of lymphedema after axillary node dissection:

A

15% to 30%

185
Q

Incidence of lymphedema after sentinel node biopsy:

A

2% to 4%

186
Q

Sensitivity and specificity of mammography:

A

90% for both

187
Q

How large must a mass be to be detected on mammography?

A

5 mm or greater

188
Q

Best time for a breast self-exam:

A

1 week after menstrual period

189
Q

General population screening recommendations for breast cancer:

A

Initial screening mammogram at age 40 and annual mammograms after age 40

190
Q

Screening recommendations for a patient at high risk for breast cancer:

A

Mammogram 10 years before the youngest age of diagnosis of breast cancer in a first- degree relative

191
Q

What percentage of breast cancers have a negative mammogram and ultrasound?

A

10%

192
Q

Suspicious findings seen on mammography for breast cancer:

A
Asymmetric density 
Distortion of architecture 
Ductal asymmetry 
Irregular borders 
Multiple clusters
Linear, small, thin, and/or branching calcifications Spiculations
193
Q

What does BIRADS stand for?

A

Breast Imaging Reporting and Data System

194
Q

What are the assemnent and recommendation for each BIRADS category?

A

BIRADS 0: Incomplete; follow-up imaging necessary
BIRADS 1: Negative; routine screening
BIRADS 2: Definite benign finding; routine screening
BIRADS 3: Probably benign; 6-month short-interval follow-up
BIRADS 4: Suspicious abnormality; biopsy should be considered
BIRADS 5: Highly suspicious ofmalignancy; appropriate action should be taken
BIRADS 6: Known biopsy-proven malignancy; ensure that treatment is completed

195
Q

Most aggressive subtype of ductal carcinoma in situ (DCIS):

A

Comedo pattern

196
Q

Risk of lymph node mets with DCIS:

A

<2%

197
Q

Surgical treatment for a <1cm low-grade DCIS:

A

Excision with 2- to 3-mm margins ± radiation

198
Q

Surgical treatment for a >1-cm DCIS:

A

Lumpectomy and radiation with 2- to 3-mm margins or total mastectomy without axillary dissection

199
Q

Indications to perform a simple mastectomy for DCIS:

A

Contraindications to radiation,

high grade, and diffuse breast involvement

200
Q

In which breast does invasive cancer arise in the setting of DCIS?

A

Usually the ipsilateral breast

201
Q

What percentage of patients get cancer in the ipsilateral breast with unresected DCIS?

A

50% to 60%

202
Q

What percentage of patients get cancer in the contralateral breast with unresected DCIS?

A

5% to 10%

203
Q

What percentage of patients develop cancer in either breast with lobular carcinoma in situ (LCIS)?

A

40%

204
Q

How much does atypical lobular hyperplasia increase the chance of developing breast cancer?

A

4-fold

205
Q

How much does atypical lobular hyperplasia in the setting of a strong family history of breast cancer increase the chance of developing breast cancer?

A

9-fold

206
Q

How much does LCIS increase the chance of developing breast cancer?

A

9-fold

207
Q

In which breast does invasive cancer arise in the setting of LCIS?

A

Carcinoma can arise in either breast

208
Q

True or False: LCIS is a premalignant lesion.

A

False; considered a marker for the development of breast cancer but not premalignant

209
Q

True or False: LCIS needs to be excised to negative margins.

A

False; negative margins are not required

210
Q

What is the most likely type of breast cancer to develop in a patient with LCIS?

A

Ductal carcinoma (70%)

211
Q

What is the percentage of finding a synchronous breast cancer at the time of diagnosis of LCIS?

A

5%

212
Q

Treatment for LCIS:

A

Close-interval follow-up, treatment with tamoxifen, or bilateral simple mastectomy

213
Q

What is the incidence of breast cancer?

A

1 in 8 women; 12% lifetime risk

214
Q

What is the breast cancer risk in a patient with no risk factors?

A

4 to 5%

215
Q

What percentage of women with breast cancer have no known risk factors?

A

75%

216
Q

Name factors that place a patient at greatly increased risk for breast cancer:

A

2 primary relatives with bilateral or premenopausal breast cancer
BRCA gene in a patient with family history of breast cancer DCIS or LCIS
Fibrocystic disease with atypical hyperplasia

217
Q

Name factors that place a patient at moderately increased risk for breast cancer:

A

Early menarche (<12 years)
Late menopause (>55)
Nulliparity (or first birth after age 30)
Environmental risk factor (high-fat diet/obesity)
Family history of breast cancer (excluding BRCA gene)
2 primary relatives with bilateral or premenopausal breast cancer
Previous breast cancer
Radiation

218
Q

What is the median survival for a patient with untreated breast cancer?

A

2 to 3 years

219
Q

What is the most common site of breast cancer?

A

Upper outer quadrant (50%)

220
Q

What is the most important prognostic staging factor for breast cancer?

A

Nodal status

221
Q

Approximate 5-year survival for a patient with breast cancer with:

A

0 positive nodes: 75%
1 to 3positive nodes: 60%
4 to 10 positive nodes: 40%

222
Q

According to AJCC cancer staging, what is a:

A

T1 breast lesion: 52 cm
T2 breast lesion: 2 to 5 cm
T3 breast lesion: >5 cm
T4 breast lesion: Skin/chest wall involvement (excluding pectoral muscles) Peau d orange: Inflammatory breast cancer

223
Q

According to AJCC cancer staging, what are N1, N2, and N3 nodal status in breast cancer?

A

N1: Ipsilateral axillary nodes
N2: Fixed ipsilateral axillary nodes
N3: lpsilateral internal mammary nodes

224
Q

According to AJCC cancer staging, what is M1 disease in breast cancer?

A

Distant metastasis, including ipsilateral supraclavicular nodes

225
Q

Most common type of breast cancer:

A

Infiltrating ductal carcinoma

226
Q

Name indications for breast biopsy:

A

Blood in cyst aspirate, bloody nipple discharge Dermatitis/ulceration of nipple
Patient concern of persistent breast abnormality
Persistent mass after aspiration
Solid mass
Suspicious lesion by imaging studies

227
Q

What are central subareolar tumors at an increased risk for?

A

These tumors are usually multicentric

228
Q

AJCC cancer staging:

A
Stage 0 breast cancer Tis, N0, M0
Stage I breast cancer T1, N0, M0
Stage Ila breast cancer
T0- to T1, N1, M0 or T2, N0, M0
Stage IIb breast cancer
T2, N1, M0 or T3, N0, M0
Stage Illa breast cancer
T0 to 3, N2, M0 or T3, N1 to N2, M0
Stage Illb breast cancer Any T4 or N3 tumors
Stage IV breast cancer M1
229
Q

How long does it take for a single malignant cell to become a 1-cm tumor?

A

5 to 7 years

230
Q

What are the sites of metastasis for breast cancer?

A

Lymph nodes (most common}
Brain
Bones
Liver Lung/pleura

231
Q

Estrogen receptor (ER)- and progesterone receptor (PR)-positive tumors are associated with:

A

Better overall prognosis and prognosis following surgery
Better response to chemotherapy and hormone manipulation
More well-differentiated tumor

232
Q

Most common place for distant metastasis for breast cancer:

A

Bone

233
Q

Name malignant tumors with a well-circumscribed, benign appearance:

A

Phyllodes tumor, medullary carcinoma, and adenoid cystic carcinoma

234
Q

What percentage of phyllodes tumors are malignant?

A

10%

235
Q

What is the treatment of a phyllodes tumor?

A

Wide local excision with 1-cm margin without axillary dissection

236
Q

Name benign conditions that can mimic breast cancer:

A

Fat necrosis, fibromatosis, granular cell tumor, radial scar

237
Q

Incidence of male breast cancer:

A

<1%of all breast cancer cases

238
Q

Risk factors for male breast cancer:

A
BRCA2 carrier, 
estrogen therapy, 
gynecomastia (from increased estrogen}, 
increased estrogen, 
Klinefelter syndrome (XXY), 
radiation
239
Q

The type of breast cancer that males develop:

A

Ductal carcinoma

240
Q

True or False: The prognosis for breast cancer is poorer in men than in women, stage for stage.

A

False; prognosis is the same stage for stage (overall prognosis is poorer secondary to men presenting at a later stage)

241
Q

Approximate risk of ovarian cancer with BRCA1 versus BRCA2 mutation:

A

40% versus 20%

242
Q

Approximate percentage of males with BRCA2 mutations that develop breast cancer:

A

6%

243
Q

How are BRCA1 and BRCA2 transmitted?

A

Autosomal dominant transmission

244
Q

True or False: The germline mutations BRCA1 and BRCA2 are completely penetrant.

A

False; they are incompletely penetrant, and some carriers may not develop cancer

245
Q

Population with the highest incidence of BRCA mutations:

A

Ashkenazi Jews (1%-3%)

246
Q

What kind of protein does the human epidermal growth factor receptor-2 (HER-2) encode for?

A

Transmembrane tyrosine kinase receptor with potent growth-stimulating activity

247
Q

Inherited breast cancer syndrome associated with an increased incidence of adrenocortical cancers, brain tumors, leukemias, soft tissue, and osteosarcomas in the same family:

A

Li-Fraumeni syndrome

248
Q

Treatment for male breast cancer:

A

Modified radical mastectomy

249
Q

Causes of male gynecomastia:

A
Medications (cimetidine, ketoconazole, spironolactone, TCAs), 
androgen insensitivity, 
gonadal failure, 
decreased testosterone, 
increased estrogen, 
illicit drugs (marijuana), 
liver failure
250
Q

Treatment for male gynecomastia:

A

Stop/change medications,
correct any hormonal imbalances/underlying cause;
if refractory to time and conservative measures, perform a biopsy versus simple mastectomy

251
Q

Define a simple/total mastectomy:

A

Excision of breast and nipple-areolar complex without removal of axillary nodes

252
Q

What is removed with a modified radical mastectomy?

A

Breast, nipple-areolar complex, and axillary nodes (level I, II)

253
Q

Potential complications after modified radical mastectomy:

A

Hematoma/seroma, infection, ipsilateral arm lymphedema, nerve injury, phantom breast syndrome, skin flap necrosis

254
Q

Considerations for prophylactic mastectomy:

A

Family history and BRCA-positive or LCIS with one of the following:
• High patient anxiety
• Lesion difficult to follow on exam/mammography
• Poor access for follow-up
• Patient preference for mastectomy

255
Q

What are indications for radiation after mastedomy?

A
Tumor>5cm
4 or more positive lymph nodes
Extracapsular nodal invasion
Fixed axillary nodes (N2) or internal mammary nodes (N3)
Inflammatory cancer Positive margins
Skin/chest wall involvement
256
Q

List complications of breast irradiation:

A

Cardiac toxicity, contralateral breast cancer, edema, erythema, pneumonitis, rib fractures, sarcoma, ulceration

257
Q

Absolute contraindications for breast-conserving therapy in invasive carcinoma:

A

An area with a history of prior therapeutic irradiation
Diffuse, malignant-appearing microcalcifications
Persistent positive margins after surgery
Pregnancy (except in third trimester, can give radiation after delivery) 2 or more primary tumors in separate quadrants of the breast

258
Q

List relative contraindications for breast-conserving therapy in invasive carcinoma:

A

Extensive gross multifocal disease in the same quadrant
History of scleroderma or active systemic lupus erythematosus
Large tumor in a small breast, if it would result in unacceptable cosmesis
Genetic predisposition to breast cancer

259
Q

What breast cancers can be treated with lumpectomy and radiation?

A

Tumors <5 cm (stage I and II)

260
Q

Chance of local recurrence after lumpectomy with radiation:

A

10%, usually within 2 years of the first operation

261
Q

Treatment for local recurrence of breast cancer after lumpedomy, axillary dissection, and radiation:

A

Salvage mastectomy

262
Q

Chance of local recurrence after mastectomy:

A

5% (4%-8%)

263
Q

What is the false-negative rate for sentinel node biopsy?

A

4%to 12%

264
Q

What are the usual chemotherapeutic regimens for breast cancer?

A

AC (Adriamycin and cyclophosphamide) followed by paclitaxel,
TAC (Adriamycin, cyclophosphamide, and docetaxel),
TC (docetaxel and cyclophosphamide), or
CMF (cyclophosphamide, methotrexate, 5-fluorouracil)

265
Q

Standard dosage of Tamoxifen with regard to adjuvant treatment for breast cancer:

A

20mg daily for 5 years

266
Q

For which patient population is Arimidex/Femara (aromatase inhibitors) useful?

A

Postmenopausal women

267
Q

Name alternative hormonal/chemotherapy options for breast cancer:

A

Aminoglutethimide, anastrozole/letrozole (aromatase inhibitors), androgenic steroid, bilateral oophorectomy, Megace

268
Q

Risk of blood clots with the use of tamoxifen:

A

1%

269
Q

Risk of endometrial cancer with the use of tamoxifen:

A

0.10%

270
Q

What is the increased relative risk of breast cancer with 5 years of hormone replacement therapy?

A

1.35 (26% increased risk)

271
Q

Recommendation for adjuvant therapy for:

A

Node negative, low-risk breast cancer:
No treatment or endocrine therapy if ER+

Node negative, higher risk ER+ breast cancer:
Chemotherapy + endocrine therapy or endocrine therapy

Node negative, higher risk ER- breast cancer:
Chemotherapy

Node positive, ER+ breast cancer in a premenopausal patient: Chemotherapy + endocrine therapy

Node positive, ER+ breast cancer in a postmenopausal patient:
Chemotherapy + endocrine therapy or endocrine therapy alone

Node positive, ER- breast cancer:
Chemotherapy

272
Q

What is the treatment ofHER-2-positive breast cancer?

A

Chemotherapy plus trastuzumab (Herceptin)

273
Q

Treatment for breast cancer in pregnancy:

A

First trimester: Modified radical mastectomy

Second trimester: Modified radical mastectomy

Third trimester: If late, can perform lumpectomy with ALND and postpartum radiation; otherwise, modified radical mastectomy

274
Q

Indications for neoadjuvant therapy:

A

Primary tumors >5 cm, fixed or matted axillary nodes, and inflammatory breast carcinoma

275
Q

Contraindications to radiation therapy:

A

Previous radiation therapy to breast or severe collagen vascular disease, pregnancy

276
Q

What is the mechanism of action of tamoxifen versus aromatase inhibitors?

A

Tamoxifen is a selective ER modulator and binds to and inhibits ER signaling in the breast.

It produces antiestrogen effects in the breast.

In postmenopausal women, the primary source of estrogen is from the conversion of androstenedione to estrone and testosterone to estradiol in peripheral tissues such as breast, skin, and adipose tissue by aromatase.

The aromatase inhibitors block this conversion, explaining why they are only effective in postmenopausal women.

277
Q

What are the side effects to aromatase inhibitors?

A

Osteoporosis, joint disorders, hypercholesterolemia

278
Q

What other cancers are associated with BRCA1 and BRCA2 mutations?

A

Prostate cancer is associated with both mutations.

BRCA1: ovarian and endometrial. Consider prophylactic TAHBSO.

BRCA2: male breast cancer, colon, pancreatic, stomach, gallbladder, melanoma

279
Q

What type of breast cancer is associated with BRCA1 and BRCA2 mutations?

A

BRCA1: poorly differentiated, receptor-negative

BRCA2: well-differentiated, receptor-positive

280
Q

What is Paget disease (of the breast)?

A

Dermatitis/scaling rash of the nipple caused by invasion of the skin by ductal carcinoma cells

281
Q

If Paget disease is confined to the nipple, how can it be treated?

A

Excision with sentinel lymph node (SLN) biopsy and radiation

282
Q

What is Stewart-Treves syndrome?

A

Lymphangiosarcoma secondary to chronic lymphedema following axillary dissection

283
Q

Term for thrombophlebitis of superficial breast veins:

A

Mondor disease

284
Q

How do you treat Mondor disease?

A

Nonsteroidal antiinflammatory drugs (NSAIDs)

285
Q

Term for a benign tumor ofthe breast found with collagen arranged in swirls and consisting of stromal overgrowth:

A

Fibroadenoma

286
Q

Term for a cellulitis/superficial infection of the breast:

A

Mastitis

287
Q

Most common bacteria to cause mastitis:

A

Staphylococcus aureus

288
Q

Treatment for mastitis:

A

Antibiotics, continue breastfeeding, heating packs, utilization of breast pumps

289
Q

Why should a patient with mastitis have dose follow-up?

A

To rule out inflammatory breast cancer

290
Q

Most common breast abnormality:

A

Accessory nipple

291
Q

Name of the syndrome associated with absence of the pectoralis muscle, amastia, hypoplasia of the chest wall, and hypoplastic shoulder?

A

Poland syndrome

292
Q

Condition in which the nipple is present but the breast mound is absent:

A

Amastia

293
Q

Most common cause of bloody discharge from the nipple:

A

lntraductal papilloma

294
Q

Treatment for an intraductal papilloma:

A

Resection (subareolar resection)

295
Q

Most common cause of brown-, green-, or straw-colored nipple discharge:

A

Fibrocystic disease

296
Q

Treatment for mastodynia:

A

Evening primrose oil, danazol, OCP

297
Q

Dose of evening primrose oil to treat mastodynia:

A

3 to 4g daily

298
Q

Most common breast lesion in adolescents and young adults:

A

Fibroadenoma

299
Q

Biopsy of a breast lesion shows prominent fibrous tissue compressing epithelial elements:

A

Fibroadenoma

300
Q

Biopsy of a breast lesion shows nests of epithelial cells invading stroma in a random fashion with the suggestion of tubule formation:

A

Infiltrating ductal carcinoma

301
Q

Biopsy of a breast lesion shows a uniform population of cells distorting, expanding, and filling the lobules:

A

LCIS

302
Q

Biopsy of a breast lesion shows an ectatic duct with a single- or double-cell layer (epithelial or myoepithelial):

A

Fibrocystic disease

303
Q

Fine-needle aspiration (FNA) of a breast lesion with the histologic appearance of poorly cohesive intact cells with variation in nuclear size, with nuclear crowding, prominent nucleoli, dumping of chromatin, and radial dispersion in a highly cellular, monomorphic pattern:

A

Breast carcinoma

304
Q

FNA of a breast lesion with the histologic appearance of broad sheets of cohesive cells with nuclei in uniform size and shape:

A

Fibroadenoma

305
Q

A 65-year-old female presented with a 2.5-cm mass in her right breast that on core needle biopsy had high-grade DCIS. She is taken to the operating room for lumpectomy and SLN biopsy. The patient was injected with both blue dye and radiotracer, but no SLN could be identified. What will this patient’s treatment indude?

A. Lumpectomy only
B. Lumpectomy and radiation only
C. Mastectomy and axillary lymph node dissection only
D. Lumpectomy and radiation and axillary lymph node dissection
E. Mastectomy and axillary lymph node dissection and radiation

A

Answer: D.

If no tracer or blue dye is taken up by the lymph nodes, a lymph node dissection must be performed along with breast-conserving therapy (i.e., lumpectomy) and radiation.

306
Q

A premenopausal 42-year-old female has been diagnosed with ER+/PR+ invasive ductal carcinoma. Her lesion is 2 cm in size, and she has 2 positive lymph nodes. She had chosen to undergo breast-conserving therapy. What will her postsurgical treatment indude?

A. Chemotherapy only
B. Radiation therapy only
C. Anastrozole only
D. Radiationtherapy and tamoxifen
E. Chemotherapy, radiation therapy, and tamoxifen
A

Answer: E.

Since the patient has positive nodal status, chemotherapy is necessary. Also undergoing breast-conserving therapy, the patient must receive radiation. Since she is premenopausal and her nodal status is positive, tamoxifen would also be given.

307
Q

A 62-year-old female presents with scaly skin surrounding her nipple that has not resolved. A biopsy including skin was performed at this area. Pathology returns with large vacuolated cells in the epithelium, and no other atypical cells or cancerous cells are present. Her treatment should be:

A. Modified radical mastectomy 
B. Simple mastectomy
C. Steroid cream
D. Simple mastectomy with SLN
E. Wide excision of the skin involved
A

Answer: B.

The patient has Paget disease that involves more than the nipple without evidence of malignancy; thus, the treatment is simple mastectomy.

308
Q

A 36-year-old female who is 20 weeks’ pregnant presents with a palpable mass. Ultrasound-guided core needle biopsy reveals a 3-cm invasive ductal carcinoma. She wishes to undergo breast-conserving therapy. What treatment is indicated for her?

A. Perform the lumpectomy and wait until postpartum for radiation therapy
B. Give neoadjuvant therapy and wait until after delivery for breast-conserving surgery and radiation
C. The patient’s condition is a contraindication to breast-conserving therapy and must undergo a modified radical mastectomy
D. Perform a lumpectomy, SLN biopsy, and radiation therapy after delivery

A

Answer: C.

Since the patient is in her second trimester, chemotherapy and radiation are contraindicated. She will need a modified radical mastectomy.

309
Q

A 47-year-old female presents with a palpable mass on physical exam. A mammogram was obtained, and her lesion was characterized as a BIRADS3. Your next step is:

A. An ultrasound
B. Excision of mass
C. Core needle biopsy
D. Repeat mammogram in 6 months, since her mammogram result was a BIRADS 3

A

Answer: C.

Even though her mammogram is BIRADS3, indicating a repeat mammogram in 6 months, the patient has a palpable mass. This alone necessitates immediate further workup with core needle biopsy.

310
Q

A 70-year-old female presents with multiple nodules on her left arm, which has been chronically swollen since her axillary lymph node dissection 7 years ago. How would you diagnose and treat this patient?

A. Fine needle biopsy and treat with radiation therapy 
B. Fine needle biopsy with wide excision
C. CT scan and chemotherapy
D. Open biopsy and wide excision
E. Open biopsy with radiation therapy
A

Answer: D.

The patient has developed Stewart-Treves syndrome (lymphangiosarcoma).

Her treatment would include an open biopsy to confirm diagnosis followed by wide excision.

311
Q

A 46-year-old female presents with a 4-cm palpable mass. Pathology and ultrasound-guided core needle biopsy reveal an invasive ductal carcinoma. The patient desires breast-conserving therapy. What treatment is indicated for her?

A. Tell her that she is not a candidate for breast-conserving therapy and requires a modified radical mastectomy
B. Neoadjuvant chemotherapy followed by breast-conserving therapy
C. Tell her that she is not a candidate for breast-conserving therapy and requires a simple mastectomy with SLN biopsy
D. Neoadjuvant chemotherapy followed by breast conservation and SLN biopsy if the tumor has an appropriate response to chemotherapy

A

Answer: D.

Given the patient’s wish for breast conservation, neoadjuvant chemotherapy can be given in an attempt to downsize the tumor.

After neoadjuvant chemotherapy, if there is an appropriate response by the tumor, breast conservation can be undertaken.

An SLN biopsy should also be performed to assess the lymph node status.

312
Q

A 64-year-old female presents with spontaneous, unilateral, bloody nipple discharge. The rest of her exam is unremarkable, and mammogram and ultrasound do not reveal any abnormalities. What is the next step in management?

A. Major duct excision
B. Reassurance, since nipple discharge is rarely associated with malignancy
C. Ductography
D. Repeat mammogram and ultrasound in 6 months

A

Answer: A.

This patient has spontaneous, unilateral, bloody nipple discharge, which is suspicious for an underlying malignancy.

The most common cause of bloody nipple discharge is a benign, intraductal papilloma.

Major duct excision is warranted because of the risk of malignancy in this patient.

Ductography has only a 60% sensitivity for malignancy, but could be a consideration in a patient who wishes to preserve breastfeeding.

313
Q

A 39-year-old female presents with a 2-cm palpable breast mass. On exam, she is also noted to have a warm, erythematous breast with peau d’orange changes. Biopsy of the mass reveals infiltrating ductal carcinoma. What is the next step in management?

A. Breast-conserving surgery with SLN biopsy
B. Modified radical mastectomy
C. Breast-conserving surgery with SLN biopsy followed by chemotherapy
D. Neoadjuvant chemotherapy followed by modified radical mastectomy
E. Neoadjuvant chemotherapy followed by breast-conserving surgery

A

Answer: D.

This patient has inflammatory breast cancer, and breast-conserving surgery is contraindicated. The management of inflammatory breast cancer is neoadjuvant chemotherapy followed by chemotherapy and radiation therapy.

314
Q

A 45-year old female after a needle biopsy of a nonpalpable breast mass reveals LCIS. What is the next step in the management of this patient?

A. Breast-conserving surgery with SLN biopsy
B. Bilateral modified radical mastectomies
C. Needle localized excisional biopsy of the lesion
D. No further surgery is indicated
E. Treat the patient with tamoxifen and close follow-up

A

Answer: C.

In this patient with LCIS and an imaging abnormality, she will need a wider biopsy to rule out an associated cancer.

A negative margin is not required at the excisional biopsy, since LCIS can be extensive.

The goal of the biopsy is to obtain a larger specimen. If a malignancy is found, it should be appropriately treated.

315
Q

A 55-year-old woman is diagnosed with Paget disease that is confined to the nipple with no associated cancer. What is the next step of her treatment?

A. Modified radical mastectomy
B. Simple mastectomy
C. Excision of the nipple-areolar area and sentinel lymph node biopsy
D. Excision of the nipple-areolar area without sentinel lymph node biopsy

A

Answer: C.

The patient has Paget disease confined to the nipple without associated cancer, so the patient can be offered breast-conserving surgery but does need a sentinel lymph node biopsy.

316
Q

How does cystosarcoma phyllodes spread in its metastasis?
A. Hematogenous
B. Lymph node
C. Bone

A

Answer: A.

Phyllodes rarely metastasizes, but if it does, it is via hematogenous spread.

It has no nodal spread; therefore, lymph node dissection is not done.

317
Q

This pattern in ductal carcinoma in situ has central necrosis seen on pathology and has a high risk for multicentricity, microinvasion, and recurrence.

A. Solid
B. Cribriform
C. Papillary
D. Comedo

A

Answer: D.

The comedo pattern of DCIS is the most aggressive subtype with areas of necrosis seen on pathology.

318
Q

Of the following subtypes of ductal carcinoma of the breast, which has the most favorable prognosis?

A. Medullary
B. Tubular
C. Mucinous(colloid)
D. Scirrhotic

A

Answer: B.

The tubular subtype has the most favorable prognosis of ductal cancer.

319
Q

Of the following subtypes of ductal carcinoma of the breast, which has the worst prognosis?

A. Medullary
B. Tubular
C. Mucinous(colloid)
D. Scirrhotic

A

Answer: D.

The scirrhotic subtype has the worst prognosis of ductal cancer.

320
Q

A 45-year-old woman was found to have microcalcifications in clusters of her left breast on screening mammography. Stereotactic guided core needle biopsy was done and revealed DCIS, ER/PR positive. Which of the following is not a treatment option for her?

A. Wide excision (lumpectomy) and radiation therapy

B. Mastectomy with sentinel lymph node biopsy

C. Radiation therapy and hormonal therapy

D. Wide excision and chemotherapy

A

D. Wide excision and chemotherapy

321
Q

A 50-year-old woman was diagnosed to have LCIS pleomorphic on core biopsy. What is your next step in management?

A. No further treatment and do usual screening

B. Excision biopsy

C. Wide excision with margins

D. Hormonal therapy

A

C. Wide excision with margins

322
Q

A 42-year-old woman presented with a recurrent breast mass after excision biopsy 2 years ago. Histology was reviewed and revealed benign phyllodes tumor. The following are true about this tumor:

A. Recurrent phyllodes tumor is always managed with mastectomy.

B. Axillary staging is necessary for malignant types.

C. Hormonaly therapy is effective is the lesion is estrogen receptor positive.

D. Locally recurrent breast mass would need chest imaging for metastatic workup.

A

D. Locally recurrent breast mass would need chest imaging for metastatic workup.

323
Q

A 55-year-old patient presented with left nipple ulcerating wound spreading to the areola. Ultrasound and mammogram showed no abnormal findings, axillary nodes are not enlarged. Which of the following is correct?

A. Consider breast MRI and tissue sampling.

B. Breast conservation is not an option.

C. Mastectomy is the best surgical management

D. Sentinel lymph node biopsy is not indicated.

A

A. Consider breast MRI and tissue sampling.

324
Q

A 32-year-old woman in her 3rd trimester was diagnosed with right breast cancer T1N1M0. Which of the following is true:

A. Neoadjuvant chemotherapy cannot be delivered.

B. Pregnancy is not a contraindication for BCS.

C. Modified radical mastectomy is the best option for her.

D. Pregnant women often present with early stage.

A

B. Pregnancy is not a contraindication for BCS.

325
Q

True about inflammatory breast cancer:

A. It is stage IIIA.

B. Preoperative systemic therapy is recommended.

C. Sentinel lymph node biopsy can be done.

D. Radiation has no role if MRM is done.

A

B. Preoperative systemic therapy is recommended.

326
Q

A 40-year-old female has enlarged axillary lymph nodes with no breast mass on ultrasound and mammogram. Which of the following is the most appropriate for her?

A. Primary consideration is TB lymphadenitis since there is no breast lesion.

B. PET scan is the best imaging procedure.

C. Do axillary lymph node dissection and radiation.

D. MRI is recommended to look for possible primary breast cancer.

A

D. MRI is recommended to look for possible primary breast cancer.

327
Q

A 49-year-old post menopausal woman has invasive ductal carincoma, 1cm in size, underwent breast conserving surgery, sentinel nodes are all negative for metastasis, ER/PR negative and HER2 positive. The following adjuvant treatment are needed except:

A. Radiation therapy

B. Targeted therapy

C. Chemotherapy

D. Hormonal therapy

A

D. Hormonal therapy

328
Q

The following are true about primary breast abscess:

A. Surgery is the only option for treatment.

B. Recurrent cases would need culture and sensitivity tests.

C. Needle aspiration is not advisable.

D. Biopsy of the cavity wall is not recommended.

A

B. Recurrent cases would need culture and sensitivity tests.

329
Q

An 18-year-old students presented with a 2.5cm palpable and movable breast mass. Ultrasound showed a hypoechoic nodule with benign features. Which of the following is true.

A. Mammogram must be done to further characterize the lesion.

B. Surgery has no role since it has benign findings.

C. Repeat ultrasound after 1 year and just monitor stability.

D. The size of the lesion can be an indication for excision biopsy.

A

D. The size of the lesion can be an indication for excision biopsy.

330
Q

A 35-year-old professional dancer presents with a well-defined, tense, smooth mass in the upper outer quadrant of the left breast. She states that the mass becomes larger just before onset of her periods. Aspiration yields a clear
yellow fluid and the mass disappears. The most
likely diagnosis is:

(A) Fibroadenoma is a cyst.

(B) Fibrocystic disease of the breast.

(C) Carcinoma in a cyst.

(D) Lipoma.

(E) Galactocele.

A
  1. (B) Breast cysts are often well demarcated and tend to get larger and contain nonbloody fluid, which is usually acellular and cytology is rarely indicated. Galactoceles present in pregnant and nursing women are filled with milky fluid.
331
Q
  1. An 18-year-old presents with a wellcircumscribed 2-cm mass in her right breast.
    The mass is painless and has a rubbery consistency and discrete borders. It appears to move
    freely through the breast tissue. What is the
    likeliest diagnosis?
    (A) Carcinoma
    (B) Cyst
    (C) Fibroadenoma
    (D) Cystosarcoma phyllodes
    (E) Intramammary lymph node
A
  1. (C) Fibroadenomas are most often found in teenage girls. They are firm in consistency, clearly defined , and very mobile. The typical feature on palpation is that they appear to move freely through the breast tissue (“breast mouse”).
332
Q
23. Galactorrhea, a milky discharge from the nipple
in nonpregnant women, is most likely to be
associated with which of the following?
(A) Fibroadenoma
(B) Tubular adenoma
(C) Pituitary adenoma
(D) Hyperparathyroidism
(E) Breast abscess
A
  1. (C) Galactorrhea is fairly common up to old age. The discharge may vary in color from brown to milky. Hormonal causes are associ- ated with elevated prolactin levels or with pitu- itary or thyroid disorders. Tranquilizers have also been implicated. Simple abscesses do not cause galactorrhea.
333
Q
  1. A 28-year-old female figure skater presents several weeks after having sustained an injury to
    her left breast. She has a painful mass in the
    upper outer quadrant. Skin retraction is noticed,
    and a hard mass, 3–4 cm in diameter, can easily
    be palpated. What is the most likely diagnosis?
    (A) Infiltrating carcinoma
    (B) Breast abscess
    (C) Hematoma
    (D) Fat necrosis
    (E) Sclerosing adenosis
A
  1. (D) Fat necrosis is a rare condition that follows injury. Diagnosis may be difficult, and mam- mography and exicision may be necessary to rule out carcinoma. Sclerosing adenosis is a variant of fibrocystic disease and may present with a hard mass. In a hematoma, evidence of resolving ecchymosis may be present.
334
Q
  1. A 35-year-old patient presents to your office
    with chronic draining subcutaneous periareolar
    abscesses, which have been incised and drained
    many times in the past 5 years but keep recurring. What is the best treatment of choice?
    (A) Repeat incision and drainage (I and D)
    since the previous procedures were
    inadequate
    (B) Long-term antibiotics
    (C) Major duct excision
    (D) Complete excision of the drainage tract
    (E) Tell the patient there is nothing to do and
    that this will eventually resolve with age
A
  1. (D) Mammary fistula also known as Zuska’s disease is felt to represent dilated laciferous ducts, which develop chronic inflammation presenting with these periareolar draining sinuses. They will continue to recur until com- pletely excised, which may require removal of the terminal duct into the nipple, leaving the wound open.
335
Q
  1. A patient presents 1 month after a benign right
    breast biopsy with a lateral subcutaneous cord
    felt just under the skin and causing pain. The
    etiology of this condition is?
    (A) Fat necrosis
    (B) Infection
    (C) Superficial thrombophlebitis
    (D) Suture granuloma
    (E) Misdiagnosed breast cancer
A
  1. (C) This entity is known as Mondor’s disease and is caused by superficial thrombophlebitis usually induced by surgery, infection, or trauma. The process is self-limiting and resolves within 2–10 weeks.
336
Q
  1. A 36-year-old woman complains of a 3-month
    history of bloody discharge from the nipple.
    At examination, a small nodule is found, deep
    to the areola. Careful palpation of the nippleareolar complex results in blood arrearing at
    the 3 O’clock position. Mammogram findings
    are normal. What is the likeliest diagnosis?
    (A) Intraductal papilloma
    (B) Breast cyst
    (C) Intraductal carcinoma
    (D) Carcinoma in situ
    (E) Fat necrosis
A
  1. (A) Intraductal papilloma is the most common cause of bloody discharge from the nipple. The lesion is treated by excision and is benign in most cases. Cancer is present in 5% of cases. Preoperative ductography can be used to help locate the offending duct .
337
Q
  1. A 35-year-old premenopausal woman whose
    mother had breast cancer comes into your office
    and has been told that she has fibrocystic breasts.
    On examination she has multiple areas of thickening but no discrete mass. Of the following
    diagnostic tests, which should be performed?
    (A) Re-examination in 6 months
    (B) Bilateral breast ultrasound
    (C) Thermography
    (D) Bilateral breast magnetic resonance
    imaging (MRI) with gadolinium
    (E) Spot compression views if an area of
    discrete asymmetry or concerning
    calcifications is seen
A
  1. (D) Patients who present with fibrocystic mas- topathy at this age should undergo routine screening mammography, either regular film or digital, and ultrasound if no obvious benign etiology is seen on mammography. Spot com- pression mammography is done for any ques- tionable abnormality. Routine use of screening MRI is not indicated at this time.
338
Q
  1. During a routine screening mammography, a
    62-year-old teacher is informed that she has
    changes on her mammography, and she should
    consult her physician. She can be reassured
    that the findings that indicate a benign condition are which of the following?
    (A) Discrete, stellate mass
    (B) Fine, clustered calcifictions
    (C) Coarse calcifications
    (D) Solid, clearly defined mass with irregular
    edges
    (E) Discrete, nonpalpable mass that has
    enlarged when compared with a mass
    shown on a mammogram taken 1 year
    previously
A
  1. (C) Coarse calicifications are usually benign. Fine, clustered califications are often milignant and require biopsy. Solid tumors of the breast, especially those that have increased in size or have changed in appearance, are suspicious for carcinoma and require biopsy.
339
Q
  1. A40-year-old lawyer comes into your office after
    seeing some information on the Internet relating
    to breast cancer. Which of the following factors
    has not shown to increase a woman’s risk for
    breast cancer?
    (A) Smoking
    (B) Previous history of benign breast
    biopsies
    (C) Atypia seen on pathology from previous
    breast biopsy
    (D) First-degree relative with history of
    breast cancer
    (E) Increasing age
A
  1. (A) Any history of previous breast biopsy, even benign, does show an increase risk of breast cancer. Atypia, family history, and increasing age also increase a woman’s risk. Smoking has not shown an increase risk for breast cancer.
340
Q

A 53-year-old waitress inquires about the implications of positive estrogen receptors (ER+) in an invasive carcinoma that is excised from her left breast.

  1. She should be informed of what?
    (A) They are more often positive in patients
    under 50 years of age.
    (B) If the receptors are positive, antiestrogen
    therapy is not indicated.
    (C) If the receptors are positive, the prognosis
    is more unfavorable.
    (D) ER and progesterone receptor (PR) status
    should be determined in all cases of
    breast carcinoma.
    (E) ER are usually negative when PR are
    positive.
A
  1. (D) ER and PR status should be determined in all cases of breast carcinoma. Positive ER and PR are indicative of an improved outlook and
    likelihood of response with antiestrogen med- ication. PR+ do not predict negative ER status.
341
Q

A 53-year-old waitress inquires about the implications of positive estrogen receptors (ER+) in an invasive carcinoma that is excised from her left breast.

  1. The patient is postmenopausal. She should be
    informed that which of the following hormonal
    therapy has been shown to be most effective?
    (A) Tamoxifen
    (B) Raloxifene
    (C) Toremifene
    (D) Megace
    (E) Aromotase inhibitors
A
  1. (E) Recent studies are showing aromatase inhibitors to be more beneficial than tamoxifen in preventing breast cancer recurrence in post- menopausal women. Tamoxifen, raloxifene, and toremifene are all selective ER modulators (SERMS), which act by competitively blocking estrogen binding sites and thus reducing esto- gen stimulation of breast tissue. Megace (mege- strol acetate) has been used for metastatic breast cancer.
342
Q
  1. A 52-year-old undergoes a left modified radical
    mastectomy for a 2-cm breast cancer. She should
    be informed that the factor which has the greatest impact on her prognosis is?
    (A) The size of the primary tumor
    (B) The histological type of the carcinoma
    (C) The number of axillary nodes positive
    for metastasis
    (D) Hormonal receptor status of the primary
    tumor
    (E) Positive findings on tests for the presence
    of the BRCA(breast cancer)1 gene
A
  1. (C) The number of positive axillary nodes remains one of the best prognostic indicators in breast carcinoma. The current American Joint Committee on Cancer (AJCC) staging classifi- cation now defines patients with 1–3 positive nodes (N1), 4–9 positive nodes (N2), and 10 or more positive nodes (N3) due to their different prognosis.
343
Q
  1. A 46-year-old woman presents with a mammogram that shows a 1-cm cluster of fine
    calcification in the right breast. Following mammographic wire localization, the lesion is
    excised and the pathology reported as ductal
    carcinoma in situ (DCIS) with comedo features
    and free margins. What advice should be given
    to the patient?
    (A) If untreated, about 30% of such lesions
    become invasive over a 10-year period.
    (B) Comedo DCIS is less aggressive than
    noncomedo DCIS.
    (C) Bilateral mastectomy and radiotherapy
    are the preferred treatments.
    (D) Axillary node dissection is always
    indicated.
    (E) Total mastectomy carries a high (50%)
    risk of carcinoma recurrence.
A
  1. (A) DCIS is a noninvasive lesion. Comedo DCIS is more aggressive than noncomedo DCIS. Axillary disease in uncommon is DCIS, and lymph node staging is generally not required. Breast conserving procedures can be performed as long as extensive or multicentric disease is not present. Radiation therapy is generally indicated after breast conserving therapy for DCIS.
344
Q
  1. A 43-year-old premenopausal patient has a
    biopsy showing focal lobular carcinoma in situ
    (LCIS) in the area of calcification. With regard
    to the LCIS, you should tell the patient which
    of the following?
    (A) She needs a simple mastectomy.
    (B) She must be placed on tamoxifen and
    chemotherapy.
    (C) This is a premalignant lesion, and she
    requires additional lumpectomy and
    radiotherapy.
    (D) She is at increased risk of breast cancer,
    and she should just be observed closely.
    (E) LCIS often presents with a mass.
A
  1. (D) She is at increased risk for breast cancer and should be followed closely. LCIS is usually in incidental finding. Although multifocal through- out both breasts, it is thought not to be precan- cerous itself but rather an indicator of increased cancer risk. Therefore, wide resection is not indi- cated. Careful examinations, every 6 months and yearly mammograms are done to detect inva- sive carcinoma at the earliest time. Lifetime breast cancer risk is about 30%.
345
Q
  1. A partially blind 65-year-old mother presents
    with a slight change in color of the areola of her
    left breast. An eczematous rash of the left areola
    has persisted for the last 3 months. Biopsy of the
    nipple reveals Paget’s disease. In Paget’s disease
    of the nipple which of the following is TRUE?
    (A) Carcinoma of the breast is rarely found.
    (B) Surgical therapy often fails to cure
    Paget’s disease.
    (C) The diagnosis should be made by nipple
    biopsy when suspected.
    (D) The underlying carcinoma when present
    is very large.
    (E) Paget’s disease of the bone is commonly
    encountered.
A
  1. (C) The diagnosis should be made by nipple biopsy when suspected. Paget’s disease repre- sents a ductal carcinoma that has grown along the ducts into the nipple/areolar region. The lesion often presents with an eczematous rash, which does not resolve and can be diagnosed with a small incisional biopsy. Typically swollen vacuolated Paget’s cells are found on histolog- icl examination. Many cases involve small breast cancers, which are missed on clinical examination and mammogram. Surgical ther- apy is often curative. This is unrelated to Paget’s disease of the bone.
346
Q
  1. A 39-year-old patient presents to your office
    with a left 3.5-cm breast tumor, which on core
    needle biopsy, is shown to be an invasive ductal
    cancer. On left axillary examination, she has a
    hard nonfixed lymph node. A biopsy of a left
    supraclavicular node is positive for malignancy. Her stage is currently classified as?
    (A) IIIC
    (B) IV
    (C) IIB
    (D) IIIB
    (E) IIA
A
  1. (A) Ipsilateral supraclavicular lymph node dis- ease is stage IIIC in breast cancer. The new AJCC staging system includes ipsilateral supr- aclavicular nodes as IIIC and not IV. These patients require appropriate metastatic workup and often get neoadjuvant chemotherapy.
347
Q
  1. A 40-year-old patient is diagnosed with a localized 1-cm infiltrating ductal cancer after a needle
    core biopsy of the lesion. She is clinical node negative; a lumpectomy and sentinel lymph node biopsy are performed. The patient develops an anaphylactic response during the case.
    Which of the following substances was the
    likely causative agent?
    (A) Fluorescein
    (B) 99 Tc radiolabeled colloid
    (C) Isosulfan blue dye
    (D) Methylene blue dye
    (E) Indigo carmine
A
  1. (C) Both methylene blue or isosulfan (lymp- hazurin) blue dye can be used for sentinel lymph node identification and have been asso- ciated with some allergic reactions. Isosulfan blue has been associated with rare anaphylac- tic reactions in <1% of patients. Methylene blue can cause skin necrosis if injected too superfi- cially. Fluorenscein and indigo carmine are not given in these surgeries. 99 Tc is given for the lymphoscintigraphy and gamma probe isola- tion of the sentinel node and has no known anaphylactic reactions.
348
Q
  1. A 65-year-old woman undergoes a lumpectomy
    and sentinel lymph node biopsy and is found to
    have a 5-mm tubular cancer ER and PR positive
    and a negative sentinel lymph node. What adjuvant treatment should be recommended?
    (A) Chemotherapy and radiation
    (B) Radiation treatment only
    (C) Hormonal therapy only
    (D) Radiotherapy and hormonal therapy
    (E) Partial breast irradiation
A
  1. (D) Generally patients with small (<1 cm) breast cancers, which are pathologically node nega- tive, are spared from chemotherapy. Radiation and hormonal therapy is indicated. Partial breast irradiation can be offered though there is no current randomized data to show the best modality of treatment (currently an ongoing NSABP/RTOG trial).
349
Q
  1. A 41-year-old patient presents to your office
    with a biopsy proven invasive ductal cancer in
    the upper outer aspect of her left breast, a suspicious palpable left axillary lymph node, and
    diffuse calcifications throughout the rest of the
    breast proven to be DCIS on stereotactic biopsy.
    The best surgical option is:
    (A) Modified radical mastectomy
    (B) Simple mastectomy
    (C) Lumpectomy with sentinel lymph node
    biopsy
    (D) Radical mastectomy
    (E) Total mastectomy with sentinel lymph
    node biopsy
A
  1. (A) This patient has a palpable axillary lymph node making sentinel node biopsy contraindi- cated. The multicentricity of the disease also makes the use of sentinel lymph node biopsy relatively contraindicated. Radical mastectomies are no longer performed unless gross tumor invasion into the pectoralis muscle is found.
350
Q
  1. A premenopausal 44-year-old woman undergoes a quadrantectomy and node dissection for
    a 2-cm infiltrating carcinoma of the left breast.
    The margins are clear, and 5 out of 15 lymph
    nodes are involved. ER and PR are positive.
    Recommended adjuvant therapy should include
    which of the following?
    (A) Radiotherapy alone
    (B) Estrogen therapy alone
    (C) Modified radical mastectomy
    (D) Chemotherapy alone
    (E) Chemotherapy, radiotherapy, and
    tamoxifen
A
  1. (E) Current National Institute of Health (NIH) consensus conference advises chemotherapy for all invasive cancers >1 cm as well as for node-positive cancers. Radiotherapy is required whenever breast conserving surgery is under- taken and tamoxifen should be given for all
    ER+ and/or PR+ invasive tumors whose patients are premenopausal.
351
Q
  1. An 18-week pregnant, 35-year-old woman
    presents after undergoing a modified radical
    mastectomy for a 2-cm ductal cancer with one
    out of fifteen positive axillary lymph nodes.
    What should she be informed of regarding
    breast cancer during pregnancy?
    (A) She cannot undergo chemotherapy until
    after she delivers.
    (B) She should have a therapeutic abortion
    in order to proceed with radiotherapy.
    (C) Breast cancer is the most common cancer
    during pregnancy.
    (D) Radiotherapy is indicated.
    (E) Most of these cancers are ER+.
A
  1. (C) Breast cancer is the most common cancer during pregnancy. It is usually ER–/PR–. Patients can undergo chemotherapy (nonmethotrexate regimens) starting after the first trimester and continue on with the pregnancy. Radiotherapy cannot be given during pregnancy, so mastec- tomy is often indicated unless the patient is toward the end of the pregnancy, and the radio- therpy can be given postpartum.
352
Q
  1. After undergoing modified radical mastectomy
    for cancer of the right breast, a 52-year-old female
    teacher becomes aware that the medial end of
    her scapula becomes prominent in protraction
    movements at the shoulder. She also complains
    of some weakness in complete abduction of the
    same shoulder. What nerve was injured?
    (A) Long thoracic
    (B) Thoracodorsal
    (C) Ulnar
    (D) Median
    (E) Intercostobrachial
A
  1. (A) Axillary dissection during modified radical mastectomy requires exposing the long tho- racic and thoracodorsal nerves. Injury to the long thoracic nerve that supplies the serratus anterior muscle causes “winging of the scapula”. The intercostobrachial nerve supplies sensory innervation to the skin in the axilla and proxi- mal upper extremity. The medial and ulnar nerves are outside of the usually axillary dis- section field.
353
Q
  1. A 50-year-old patient has recently undergone a
    mastectomy for a 2.5-cm multicentric breast
    cancer with three positive axillary lymph nodes
    (stage IIB). A metastatic survey is done, and is
    negative, and she receives adjuvant chemotherapy. The most common site for distant metastasis would be:
    (A) Brain
    (B) Bone
    (C) Lung
    (D) Gastrointestinal tract
    (E) Liver
A
  1. (B) Bone metastasis is the most common distant metastatic site for breast cancer. They are typi- cally osteolytic lesions and can be treated by biphosphonates, which inhibit bone deminer- alization and have been shown to reduce the pathologic fracture frequency and need of radiation.
354
Q
  1. A45-year-old premenopausal woman undergoes
    a left breast lumpectomy for a 1.5-cm, lymph
    node positive, hormone sensitive invasive breast
    cancer. She receives chemotherapy, radiotherapy,
    and is on tamoxifen. Recommended follow-up
    after therapy should always include:
    (A) Blood tumor markers drawn every
    3–6 months after treatment.
    (B) Routine monitoring of liver function
    tests (LFTs) every 3–6 months after
    treatment.
    (C) Yearly bone scans.
    (D) Routine clinical examination every
    3–6 months for the first 5 years after
    treatment as well as continued yearly
    mammography.
    (E) Yearly breast MRI with gadolinium.
A
  1. (D) Follow-up after breast cancer treatment is very variable. There is no consensus and no follow-up test has shown a survival advantage. Routine 3–6 month clinical examinations and yearly mammography should always be per- formed. The use of tumor markers such as a CA 15-3 has not shown any proven significant value and may lead to unnecessary worry.
355
Q
  1. A 43-year-old female requests breast augmentation surgery. She has no family history of
    breast cancer and her clinical examination fails
    to reveal any evidence of pathology. What
    should she be informed about the procedure?
    (A) In the United States only silicone
    gel-filled and not saline-filled implants
    are performed.
    (B) Breast implants increase the incidence of
    malignancy of the breast.
    (C) The occurrence of subsequent breast
    cancer occurs at a later stage than those
    without implants.
    (D) Saline implants have a more natural
    appearance than silicone gel-filled
    implants.
    (E) Implants in the submuscular plane
    allow better mammographic findings
    than those placed in the subglandular
    position.
A
  1. (E) There is no evidence that long-term insertion of breast implants leads to an increased inci- dence of breast cancer or detection of the cancer at an inappropriate late stage. Although the use of silicone gel implants is still confined by the Food and Drug Administration (FDA) to select circumstances (e.g., breast reconstruction fol- lowing mastectomy), retrospective studies to date have failed to demonstrate a significant increase in the incidence of collagen disease in patients who have had a silicone breast implant.
356
Q
  1. A 56-year-old male patient develops an accentric hard breast lump over the past few months
    and a biopsy proves this to be breast carcinoma.
    Of all breast cancers, the rate of occurrence in
    males is which of the following?
    (A) <1%
    (B) 4%
    (C) 7%
    (D) 10%
    (E) >10%
A
  1. (A) Cancer of the breast in males constitutes <1% of total cases. It tends to present at a more advanced stage in men than in women, because it is often overlooked. It may easily be confused with the more commonly occurring condition of gynecomastia. Careful clinical radiological follow-up studies are indicated.
357
Q
  1. A 25-year-old nonalcoholic man has noticeable
    right gynecomastia since age 20. He is most
    uncomfortable and reluctant to swim or exercise at a gym for fear of being an object of derision. He should be advised to have which of
    the following?
    (A) Right mastectomy
    (B) Observation
    (C) Needle biopsy of the breast
    (D) Endocrine workup and right
    subcutaneous mastectomy
    (E) Testosterone therapy by transdermal
    patch
A
  1. (D) In general, persistent gynecomastia should be evaluated to rule out endocrine abnormalities. In most cases, none are found. Subcutaneous astectomy is indicated if the patient is self- conscious.
358
Q
  1. A36-year-old woman presents with a substantial
    unilateral breast enlargement. She had presumed
    that this was normal, but on examination, a large,
    firm tumor is palpated by the attending physician. There is early erosion on the skin. A favorable outlook can be anticipated if the lesion is
    which of the following?
    (A) Sarcoma
    (B) Cystosarcoma phyllodes
    (C) Colloid carcinoma
    (D) Infiltrating carcinoma
    (E) Inflammatory carcinoma
A
  1. (B) Cystosarcoma phyllodes is a tumor that is very slow growing and has a good prognosis if treated by mastectomy. It is characterized by large polygonal cells with abundant cytoplasm and lymphoid infiltration.
359
Q
  1. A 55-year-old postmenopausal woman undergoes a left axillary lymph node biopsy, which
    turns out to be an adenocarcinoma. Breast
    examination fails to show any abnormality
    and mammography, ultrasound, and metastatic workups are all negative. The tumor is
    ER+/PR+. The following statements are true
    EXCEPT for which of the following?
    (A) Recurrence and survival results for this
    patient are worse than those identified
    with primary tumor in the breast.
    (B) This is most likely a lesion from the
    larynx or pharynx.
    (C) This is a common site for papillary
    carcinoma of the thyroid to metastasize.
    (D) The treatment should be a left axillary
    dissection followed by chemotherapy
    and radiation therapy.
    (E) A primary breast cancer is only found in
    10–20% of mastectomy specimens.
A
  1. (D) Occult primary breast cancer is a rare but well-known entity. Stage for stage these patients have a similar prognosis as other patients with node-positive breast cancer. The primary tumor
    is often found in the breast (60–70% of mastec- tomy specimens). Either modified radical mas- tectomy and chemotherapy or just axillary dissection with radiation and chemotherapy are accepted treatment choices.
360
Q

DIRECTIONS (Questions 51 through 54): Each set
of matching questions in this section consists of a
list of lettered options followed by several numbered items. For each numbered item, select the
appropriate lettered option. Each lettered option
may be selected once.
Questions 51 to 54
(A) Tubular
(B) Medullary
(C) Colloid
(D) Inflammatory carcinoma
(E) Infiltrating ductal carcinoma
(F) Infiltrating lobular carcinoma
(G) DCIS
(H) LCIS
(I) Paget’s disease

  1. The histology that shows an intense lymphoplasmacytic reaction around and within the
    tumor, which is usually poorly differentiated
    with a high mitotic rate. SELECT ONE.
A
  1. (B) Medullary tumors have an intense sur- rounding lymphoid reaction and though poorly differentiated, have a favorable prognosis com- pared to other invasive cancers. They tend to be hormone receptor negative.
361
Q

DIRECTIONS (Questions 51 through 54): Each set
of matching questions in this section consists of a
list of lettered options followed by several numbered items. For each numbered item, select the
appropriate lettered option. Each lettered option
may be selected once.
Questions 51 to 54
(A) Tubular
(B) Medullary
(C) Colloid
(D) Inflammatory carcinoma
(E) Infiltrating ductal carcinoma
(F) Infiltrating lobular carcinoma
(G) DCIS
(H) LCIS
(I) Paget’s disease

  1. Microscopic examination of this malignancy
    shows large vacuolated cells. SELECT ONE.
A
  1. (I) Paget’s disease is characterized by these large vacuolated intradermoid cells usually arising from a ductal carcinoma that is thought to have grown along the duct to the nipple.
362
Q

DIRECTIONS (Questions 51 through 54): Each set
of matching questions in this section consists of a
list of lettered options followed by several numbered items. For each numbered item, select the
appropriate lettered option. Each lettered option
may be selected once.
Questions 51 to 54
(A) Tubular
(B) Medullary
(C) Colloid
(D) Inflammatory carcinoma
(E) Infiltrating ductal carcinoma
(F) Infiltrating lobular carcinoma
(G) DCIS
(H) LCIS
(I) Paget’s disease

  1. Clinical findings of this breast cancer typically
    includes a rash-like erythema, which spreads
    throughout the skin of the breast. SELECT ONE.
A
  1. (D) Inflammatory breast cancer is a very aggres- sive form of breast cancer characterized by intradermal lymphatic spread of tumor.
363
Q

DIRECTIONS (Questions 51 through 54): Each set
of matching questions in this section consists of a
list of lettered options followed by several numbered items. For each numbered item, select the
appropriate lettered option. Each lettered option
may be selected once.
Questions 51 to 54
(A) Tubular
(B) Medullary
(C) Colloid
(D) Inflammatory carcinoma
(E) Infiltrating ductal carcinoma
(F) Infiltrating lobular carcinoma
(G) DCIS
(H) LCIS
(I) Paget’s disease

  1. This histologic variant is characterized by a
    linear (“indian-file”) arrangement of tumor
    cells and a tendency to grow circumferentially
    around ducts and lobules. SELECT ONE.
A
  1. (F) Infiltrating lobular cancers have this typical linear (“indian-file”) arrangement of cells. There is a higher incidence of multifocality and bilat- erality with lobular cancers. These cancers have a greater tendency to be hormone sensitive.